CRUXv 24 N 7

Download as pdf or txt
Download as pdf or txt
You are on page 1of 64

385

THE OLYMPIAD CORNER


No. 193
R.E. Woodrow
All communications about this column should be sent to Professor R.E.
Woodrow, Department of Mathematics and Statistics, University of Calgary,
Calgary, Alberta, Canada. T2N 1N4.
As a rst Olympiad to give you puzzling pleasure, we give the 18th
Austrian-Polish Mathematics Competition written in Austria June 28{30,
1995. My thanks go to Bill Sands, University of Calgary, who collected this
contest while assisting at the International Olympiad in Toronto in 1995, as
well as to Walther Janous, Ursulinengymnasium, Innsbruck, Austria.
18th AUSTRIAN-POLISH MATHEMATICS
COMPETITION
Problems of the Individual Contest
June 28-29, 1995 (Time: 4.5 hours)
1. For a given integer n  3 nd all solutions (a1; : : : ; an) of the
system of equations
a3 = a2 + a1 ; a4 = a3 + a2; : : : ; an = an,1 + an,2
a1 = an + an,1 ; a2 = a1 + an
in real numbers.
2. Let A1; A2; A3; A4 be four distinct points in the plane and let
X = fA1; A2; A3; A4g. Show that there exists a subset Y of the set X
with the following property: there is no disc K such that K \ X = Y .
Note: All points of the circle limiting a disc are considered to belong to the
disc.
3. Let P (x) = x4 + x3 + x2 + x + 1. Show that there exist polyno-
mials Q(y ) and R(y ) of positive degrees, with integer coecients, such that
Q(y)  R(y) = P (5y2) for all y.
4. Determine all polynomials P (x) with real coecients, such that
(P (x))2 + (P (1=x))2 = P (x2 )P (1=x2) for all x 6= 0:
5. An equilateral triangle ABC is given. Denote the mid-points of
sides BC , CA, AB respectively by A1 , B1 , C1 . Three distinct parallel lines
p; q;r are drawn through A1, B1, C1, respectively. Line p cuts B1C1 at A2;
386

line q cuts C1A1 at B2 ; line r cuts A1 B1 at C2 . Prove that the lines AA2 ,
BB2, CC2 concur at a point D lying on the circumcircle of triangle ABC .
6. The Alpine Club consisting of n members organizes four high-
mountain expeditions for its members. Let E1 , E2, E3, E4 be the four teams
participating in these expeditions. How many ways are there to compose
those teams, given the condition that E1 \E2 6= ;, E2 \E3 6= ;, E3 \E4 6= ;?
Problems of the Team Contest
June 30, 1995 (Time: 4 hours)
7. For every integer c consider the equation 3y4 + 4cy3 + 2xy +
48 = 0, with integer unknowns x and y. Determine all integers c for which
the number of solutions (x; y ) in pairs of integers satisfying the additional
conditions (A) and (B) is a maximum:
(A) the number jxj is the square of an integer;
(B) the number y is square-free (that is, there is no prime p with p2
dividing y ).
8. Consider the cube with vertices f1; 1; 1); that is, the set
f(x; y; z) : jxj  1; jyj  1; jzj  1g. Let V1; : : : ; V95 be points of that
cube. Denote by vi the vector from (0; 0; 0) to Vi. Consider the 295 vectors
of the form s1v1 + s2 v2 +    + s95 v95, where si = 1 or si = ,1.
(a) Let d = 48. Show that among all such vectors one can nd a vector
w = (a; b; c) with a2 + b2 + c2  d.
(b) Find a number d < 48 with the same property.
Note: The smaller d, the better mark will be attracted by the solution.
9. Prove that the following inequality holds for all integers n;m  1
and all positive real numbers x; y :
(n , 1)(m , 1)(xn+m + yn+m ) + (n + m , 1)(xn ym + xm y n)
 nm(xn+m,1y + xyn+m,1):

The next contest we give was also collected by Bill Sands while he
was assisting at the IMO in Toronto. These are the problems of the 9th
Iberoamerican Mathematical Olympiad held September 20, 21 in Fortaleza,
Brazil. Students were given 4 12 hours each day.
9th IBEROAMERICAN MATHEMATICAL OLYMPIAD
Fortaleza, Brazil, September 20{21, 1994
First Day | Time: 4.5 hours
1. (Mexico): A natural number n is called brazilian if there exists an
integer r, with 1 < r < n , 1, such that the representation of the number
387

n in base r has all the digits equal. For example, 62 and 15 are brazilian,
because 62 is written 222 in base 5 and 15 is 33 in base 4. Prove that 1993
is not brazilian, but 1994 is brazilian.
2. (Brazil): Let ABCD be a cyclic quadrilateral. We suppose that
there exists a circle with centre in AB , tangent to the other sides of the
quadrilateral.
(i) Show that AB = AD + BC .
(ii) Calculate, in terms of x = AB and y = CD, the maximal area that
such a quadrilateral can reach.
3. (Brazil): In each cell of an n  n chessboard is a lamp. When a lamp
is touched, the state of this lamp, and also the state of all the lamps in its
row and in its column, is changed (switched from OFF to ON and vice versa).
At the beginning, all the lamps are OFF. Show that it is always possible, with
suitable sequence of touches, to turn ON all the lamps of the chessboard,
and nd, in terms of n, the minimal number of touches in order that all the
lamps of the chessboard are ON.
Second Day | Time: 4.5 hours
4. (Brazil): The triangle ABC is acute, with circumcircle k. Let P be
an internal point to k. The lines AP , BP , CP meet k again at X , Y , Z .
Determine the point P for which triangle XY Z is equilateral.
5. (Brazil): Let n and r be two positive integers. We wish to con-
struct r subsets of f0; 1; : : : ; n , 1g, called A1 ; : : : ; Ar , with card(Ai) = k
and such that, for each integer x, 0  x  n , 1, there exist x1 2 A1 ;
x2 2 A2; : : : ; xr 2 Ar (an element in each subset), with
x = x1 + x2 +    + xr :
Find, in terms of n and r, the minimal value of k.
6. (Brazil): Show that all natural numbers n  21000000 can be
obtained beginning at 1 with less than 1100000 sums; that is, there exists
a nite sequence of natural numbers x0 ; x1 ; : : : ; xk , with k < 1100000,
x0 = 1, xk = n, such that for each i = 1; 2; : : : ; k, there exists r, s, with
0  r < i, 0  s < i, and xi = xr + xs .

As a nal problem set to challenge you, we present the problems of the


IX, X and XI Grade of the Georgian Mathematical Olympiad, Final Round
for 1995. It is interesting that 60% of the Grade XI problems come from the
Grade IX paper. My thanks again go to Bill Sands, University of Calgary, for
collecting these problems while he assisted with the IMO in Toronto in 1995.
388

GEORGIAN MATHEMATICAL OLYMPIAD 1995


Final Round
GRADE IX
1. A three-digit number was decreased by the sum of its digits. Then
the result was decreased by the sum of its digits and so on. Show that on
the 100th step of this procedure the result will be zero, whatever the initial
three-digit number is chosen. How many repetitions are enough to get zero?
2. Two circles of the same size are given. Seven arcs, each of them
of 3 measure, are taken on the rst circle and 10 arcs, each of them of 2
measure, are taken on the second one. Prove that it is possible to place one
circle on the other so that these arcs do not intersect. Is it or is it not possible
to prove the same if the number of arcs with measure 2 is 11?
3. Prove that if the product of three positive numbers is 1 and their
sum is more than the sum of their reciprocals, then only one of these numbers
can be more than 1.
4. Prove that in any convex hexagon there exists a diagonal which cuts
from the hexagon a triangle with area less than 16 of the area of the hexagon.
5. The set M of integers has the following property: if the numbers
a and b are in M , then a + 2b also belongs to M . It is known that the set
contains positive as well as negative numbers. Prove that if the numbers a,
b and c are in M , then a + b , c is also in M .
GRADE X
1. (a) Five di erent numbers are written in one line. Is it always pos-
sible to choose three of them placed in increasing or decreasing order?
(b) Is it always possible to do the same, if we have to choose four num-
bers from nine?
2. (Same as IX.2)
3. Prove that for any natural number n, the average of all its factors
lies between the numbers pn and n+1 2 .

4. The incircle of a triangle divides one of its medians into three equal
parts. Find the ratio of the sides of the triangle.
5. The function f is given on the segment [0; 1]. It is known that
f (x)  0 and f (1) = 1. Besides that, for any two numbers x1 and x2, if
x1  0, x2  0 and x1 + x2  1, then f (x1 + x2 )  f (x1) + f (x2).
(a) Prove that f (x)  2x for any x.
(b) Does the inequality f (x)  1:9x hold for every x?
389

GRADE XI

1. (Same as IX.3)
2. (Same as IX.1)
3. How many solutions has the equation x = 1995 sin x + 199?
4. (Same as IX.4)
5. A natural number is written in each square of an m  n rectangular
table. By one move, it is allowed to double all numbers of any row or subtract
1 from all numbers of any column. Prove that by repeating these moves
several times, all numbers in the table become zeros.

Next a bit of housekeeping. After the columns were set, and before they
appeared in print form, I received solutions from Pavlos Maragoudakis to the
six problems of the Swedish contest for which we published the solutions last
issue [1997: 196; 1998: 328{329]. He also submitted solutions to problems
1, 2, 3 and 5 of the Dutch Mathematical Olympiad, Second Round, 1993
[1997: 197; 1998: 329{332]. Last issue we gave solutions to all but the last
problem. It was rather unfortunate timing in terms of acknowledging his
contribution, but we are able to close out the le by having a complete set
of solutions from the readers.
5. P1; P2; : : : ; P11 are eleven distinct points on a line. PiPj  1 for
every pair Pi , Pj . Prove that the sum of all (55) distances Pi Pj , 1  i <
j  11 is smaller than 30.
Solution by Pavlos Maragoudakis, Pireas, Greece.
Without loss of generality we suppose that P1 ; P2 ; : : : ; P11 are adja-
cent.

P1 P2 qqq P11

Now if 1  i < j  11, then Pi Pj = Pj P1 , PiP1 . So


X X
PiPj = (Pj P1 , PiP1 )
1i<j 11 1i<j 11
= 10P11P1 + 9P10 P1 , P10 P1 + 8P9 P1 , 2P9P1
+7P8 P1 , 3P8 P1 + 6P7 P1 , 4P7P1 + 5P6P1
,5P6P1 + 4P5P1 , 6P5P1 + 3P4P1 , 7P4P1
+2P3 P1 , 8P3 P1 + P2 P1 , 9P2 P1
390

= 10P11P1 + 8P10P1 + 6P9 P1 + 4P8P1 + 2P7P1


,2P5P1 , 4P4P1 , 6P3P1 , 8P2P1
= 10P11P1 + 8(P10P1 , P2P1) + 6(P9P1 , P3 P1 )
+4(P8P1 , P4 P1 ) + 2(P7P1 , P5P1)
= 10P11P1 + 8P10P2 + 6P9 P3 + 4P8P4 + 2P7P5
< 10  1 + 8  1 + 6  1 + 4  1 + 2  1 = 30

Also setting the record straight, I found amongst the solutions for an-
other contest, the solution by Miguel Amengual Covas, Cala Figuera, Mal-
lorca, Spain to problem 2 of the Dutch Mathematical Olympiad, Second
Round, for which we published a solution last issue [1997: 197, 1998: 330{
331]. My apologies.
While we do not normally give solutions to problems of the USAMO,
I am giving two comments/solutions from our readers to problems of the
USAMO 1997 [1997: 261, 262].
2. Let ABC be a triangle, and draw isosceles triangles BCD, CAE,
ABF externally to ABC , with BC; CA; AB as their respective bases. ! !Prove !
that the lines through A; B; C perpendicular to the lines EF; FD; DE ,
respectively, are concurrent.
Comment by Mansur Boase, student, St. Paul's School, London, Eng-
land.
The result is immediate from Steiner's Theorem:
If the perpendiculars from the vertices A, B , C of a triangle ABC to the
sides B1 C1 , C1 A1 , and A1 B1 , respectively, of a second triangle A1 ; B1 ; C1
are concurrent, then the perpendiculars from the vertices A1 ; B1 ; C1 of the
triangle A1 B1C1 to the sides BC , CA, AB are also concurrent.
5. Prove that for positive real numbers a, b, c,
(a3 + b3 + abc),1 + (b3 + c3 + abc),1 + (c3 + a3 + abc),1  (abc),1:
Solutions by Mansur Boase, student, St. Paul's School, London, Eng-
land and by Murray S. Klamkin, University of Alberta, Edmonton, Alberta.
We give Klamkin's presentation.
Since the inequality is homogeneous, we can assume abc = 1. Then if
we let x = a3 , y = b3, z = c3 , the inequality becomes
1 1 + 1
1+x+y 1+y+z 1+z+x  1
+ (1)
where xyz = 1 and x, y , z are positive. On expanding, (1) is equivalent to
(x + y + z)(xy + yz + zx , 2)  3 :
391

This follows from the known elementary inequalities


x + y + z   yz + zx + xy 1=2  (xyz)1=3 :
3 3
There is equality if and only if x = y = z = 1.
Comment: The inequality in the form (1) was also given in the Spring
1997, Senior A-Level Tournament Of The Towns competition. A generaliza-
tion to
1 1    1
+
1 + S , x1 1 + S , x2 + + 1 + S , xn  1 ;
where S = x1 + x2 +    + xn , x1 x2 : : : xn = 1, and xi > 0 is due to Dragos
Hrimiuc, University of Alberta, and will probably appear as a problem in
Math. Magazine.

Next we give two solutions by our readers to two problems of the 3rd
Ukrainian Mathematical Olympiad, March 26{27, 1994 given in [1997: 262].
2. (9{10) A convex polygon and point O inside it are given. Prove that
for any n ,,!
> 1 there ! exist points A1 ; A2 ;, : : : ; An on the sides of the polygon
such that OA1 + ,, OA2 + : : : + ,,! OAn = ! 0.
Solution by Murray S. Klamkin, University of Alberta, Edmonton, Al-
berta.
It follows by continuity that there always exists a chord A1 OA01 such
that A1 O = A01 O and hence ,, OA !1 + ,,!
OA 01 = , !
0 . Similarly, there exists
a chord A2 A03 which is bisected by the midpoint O1 of OA01 . It follows by
the parallelogram law that ,,
,,! OA!2 + ,,!OA03 = ,,! OA01 and hence ,, OA!1 + ,,OA!2 +
OA03 = ,! 0 . Again similarly there exists a chord A3 A04 which is bisected by
the midpoint of OA03 so that ,, OA!1 + ,,
OA!2 + ,, OA!3 + ,,!
OA04 = ,! 0 , and so on
for any number of vectors n > 1.
3. (10) A sequence of natural numbers ak , k  1, such that for each k,
ak < ak+1 < ak + 1993 is given. Let all prime divisors of ak be written for
every k. Prove that we receive an in nite number of di erent prime numbers.
Solution by Pavlos Maragoudakis, Pireas, Greece.
We suppose that there is a sequence of natural numbers such that
ak < ak+1 < ak + 1993, k  1, and the set of all prime divisors of all
ak is nite. Let p1a;1p2; : : a: ; pr list all the prime divisors of all ak . Now every
ak has the form p1 : : : pr r , ai = 0; 1; 2; : : : , i = 1; : : : ; r.
Let S = fpa1 1 pa2 2 : : : par r j ai = 0; 1; 2; : : : ; i = 1; 2; : : : ; rg:
De ne (xn) with x1 < x2 < : : : such that S = fxn = n 2 N g. We
have that ak < ak+1 and ak 2 S , k  1. Thus (ak ) is a subsequence of
392

(xk ). But ak < ak,1 + 1993 < ak,2 + 2  1993 <    < a1 + (k , 1)1993 <
k(a1 + 1993); k  1.
Hence ak < k(a1 + 1993), k  1. Therefore
X1 1 X1 1 1 X1 1
> = a + 1993 n = +1
n=1 an n=1 n(a1 + 1993) 1 n=1
1
X 1 1
1  X X 1
and
n=1 na x =
n=1 n
1 pa2 : : : pa4
a1 ;:::;ar 0 p1 2 r
1 1 !
X 1
X 1
!
1
X 1
!
= a2 : : :
a1 =0 p1 a2 =0 p2 r =0 pr
1 ar
= p p,1 1  p p,2 1    p p,4 1 < +1 ;
1 2 4
which is a contradiction.

We now turn our attention to the solutions by readers to problems of


the Mock Test of the Hong Kong Committee for the IMO 1994 [1997: 322{
323].
INTERNATIONAL MATHEMATICAL OLYMPIAD
1994
Hong Kong Committee | Mock Test, Part I
Time: 4.5 hours
1. In a triangle 4ABC , \C = 2\B. P is a point in the interior of
4ABC satisfying that AP = AC and PB = PC . Show that AP trisects
the angle \A.
Solutions by Miguel Amengual Covas, Cala Figuera, Mallorca, Spain;
and by D.J. Smeenk, Zaltbommel, the Netherlands. We give the solution by
Amengual Covas.
Let \PAC and \BAP be 2 and respectively. Then, since
\C = 2\B , we deduce from A + B + C = 180 that
2 + + 3B = 180: (1)
A

P
B C
393

The angles at the base of the isosceles triangle PAC are each 90 , . Also
4BPC is isosceles, having base angles
C , (90 , ) = 2B + , 90;
and so
\BPA = 180 , (\PBA + \BAP )
= 180 , [B , (2B + , 90 ) + 180 , 2 , 3B ]
= 4B + 3 , 90 :
As usual, let a, b and c denote the lengths of the sides BC , AC and AB .
By the Law of Cosines, applied to 4BPA, where PA = b and
PB = PC = 2b sin ,
c2 = b2 + (2b sin )2 , 2  b  2b sin  cos(4B + 3 , 90) ;
so that
c2 = b2[1 + 4 sin2 , 4 sin sin(4B + 3 )] : (2)
We now use the fact that \C = 2\B is equivalent to the condition
c2 = b(b + a), which has appeared before in CRUX [1976: 74], [1984:
278] and [1996: 265{267]. Since a = 2  PC  cos(2B + , 90 ) =
4b sin sin(2B + ), we have
c2 = b2[1 + 4 sin sin(2B + )] : (3)
Therefore, from (2) and (3), we get
b2[1 + 4 sin2 , 4 sin sin(4B + 3 )] = b2[1 + 4 sin sin(2B + )] ;
which simpli es to
sin , sin(4B + 3 ) = sin(2B + ) :
Since sin , sin(4B + 3 ) = ,2 cos(2B + 2 ) sin(2B + ), this equation
may be rewritten as
sin(2B + )  [1 + 2 cos(2B + 2 )] = 0:
Since, from (1), 2B + < 180 , we must have 1+2 cos(2B +2 ) = 0,
giving cos(2B + 2 ) = ,1=2 ; that is,
2B + 2 = 120 (4)
since, again from (1), 2B + 2 < 180 .
Finally, we may eliminate B between (1) and (4) to obtain = . The
result follows.
394

Mock Test, Part II


Time: 4.5 hours

1. Suppose that yz + zx + xy = 1 and x, y, and z  0. Prove that


p
x(1 , y )(1 , z ) + y(1 , z )(1 , x ) + z(1 , x )(1 , y )  4 9 3 :
2 2 2 2 2 2

Solutions by Murray S. Klamkin, University of Alberta, Edmonton,


Alberta; and Pavlos Maragoudakis, Pireas, Greece. We give the solution
by Klamkin.
We rst convert the inequality to the following equivalent homoge-
neous one:
x(T2 , y2)(T2 , z2) + y(T2 , z2)(T2 , x2) +pz(T2 , x2)(T2 , y2)
 (4 3=9)(T2)5=2
where T2 = yz + zx + xy , and for subsequent use T1 = x + y + z , T3 = xyz .
Expanding out, we get
T1 T22 , T2
X p
x(y2 + z2) + T2T3  (4 3=9)(T2)5=2 ;
or
p
T1 T22 , T2(T1T2 , 3T3) + T2T3 = 4T2T3  (4 3=9)(T2)5=2 :
Squaring, we get one of the known Maclaurin inequalities for symmetric
functions: p p
3T  2 T2=3 :
3

There is equality if and only if x = y = z .

To nish this number of the Corner we give two solutions to problems


of the 45th Mathematical Olympiad in Poland, Final Round [1997: 323{324].
1. Determine all triples of positive rational numbers (x; y; z) such that
x + y + z, x,1 + y,1 + z,1 and xyz are integers.
Solution by Murray S. Klamkin, University of Alberta, Edmonton,
Alberta.
Let x + y + z = n1, x1 + y1 + z1 = n2, and xyz = n3 , where n1, n2, n3
are integers. Then yz + zx + xy = n2n3 and x, y , z are roots of the cubic
t3 , n1t2 + n2n3t , n3 = 0 :
395

As known, the only rational roots of the latter are factors of n3, and
consequently x, y , z are integers.
The only triples of integers (x; y;z ), aside from permutations, which
satisfy x1 + y1 + 1z = n2 are
(1; 1; 1); (1; 2; 2); (2; 3; 6); (2; 4; 4); and (3; 3; 3):
5. Let A1; A2; : : : ; A8 be the vertices of a parallelepiped and let O be
its centre. Show that
4(OA21 + OA22 +    + OA28 )  (OA1 + OA2 +    + OA8 )2 :
Solution by Murray S. Klamkin, University of Alberta, Edmonton,
Alberta.
Let one of the vertices be the origin and let the vectors B + C ,
C + A, A + B denote the three coterminal edges emanating from this origin.
Then the vectors to the remaining four vertices are S + A, S + B , S + C ,
and 2S where S = A + B + C and which is also the vector to the centre.
The inequality now becomes
2(S2 + A2 + B 2 + C 2)  (jS j + jAj + jB j + jC j)2 ;
or
S2 + A2 + B2 + C 2  2jSjfjAj + jBj + jC jg +2fjBj jC j + jC j jAj + jAj jBjg:
Since
S2 = A2 + B2 + C 2 + 2B  C + 2C  A + 2A  B;
the inequality now becomes
S2 ,B C ,C A,AB  jSjfjAj+jBj+jC jg+fjBj jC j+jC j jAj+jAj jBjg:
Clearly,
S2  jSjfjAj + jBj + jC jg
and
,B  C , C  A , A  B  jBj jC j jAj + jAj jBj :
There is equality if and only if the parellelepiped is degenerate,
for example, B = C = O.

That completes this number of the Olympiad Corner. Send me your


nice solutions and Olympiad contests.
396

BOOK REVIEWS
Edited by ANDY LIU
Dissections : Plane & Fancy
by Greg N. Frederickson,
published by Cambridge University Press, 1997,
ISBN# 0-521-57197-9, hardcover, 310+ pages, $34.95.
Reviewed by Andy Liu, University of Alberta.
This is a much awaited sequel to Harry Lindgren's 1964 classic work,
Geometric Dissections, which the author (G.N.F.) revised and augmented in
the 1972 Dover edition. Actually, the current volume is much more than just
a sequel. It is the most comprehensive treatise on the subject of geometric
dissections. It may be enjoyed on at least three levels.
First and foremost, this book is a collection of interesting dissection
puzzles, old and new. Only some background in high school geometry is
needed to fully enjoy these problems. Can you cut an octagon into ve pieces
and rearrange them into a square? How about turning a star into a pentagon?
The solutions, which are both appealing but for somewhat opposing reasons,
are shown below.

Octagon to square

Star to pentagon
This book is also an instructive manual on the art and science of geo-
metric dissections. While one may admire the ingenuity which produced
the spectacular solutions, the author probes into the underlying fabric which
might have led to such incisive insight. Many techniques are discussed, too
many to enumerate here. A favourite is that of tessellation. Below are two
tilings which might have suggested the dissections above.
397

Tilings for octagon, square

Cut the star for tiling

Tilings for star, pentagon


Finally, this book is an important historical document, detailing the
inter-cultural development of the subject. Travel from the palace school of
tenth-century Baghdad to the mathematical puzzle columns in turn-of-the-
century newspapers, from the 1900 Paris Congress of Mathematicians to the
night sky of Canberra. Readers puzzled by this quote need look no further
than the illustrious names of Abu'l Wafa, Henry Dudeney/Sam Loyd, David
Hilbert/Max Dehn and Harry Lindgren. Biographical sketches of Wafa, Du-
deney, Loyd and Lindgren are provided, along with those of over forty other
people who have made signi cant contributions to geometric dissections.
The writing style is very engaging, and the book is good reading even if one
skips over some of the more complicated technical details.
In conclusion, the reviewer echoes Martin Gardner that this book will
be a classic. It comes with the highest recommendation.
398

THE SKOLIAD CORNER


No. 33
R.E. Woodrow
As a contest this issue we give the Senior High School Mathematics
Contest, Preliminary Round 1998 of the British Columbia Colleges. My thanks
go to the organizer, Jim Totten, The University College of the Cariboo, for
forwarding me the contest materials. Time allowed is 45 minutes!
BRITISH COLUMBIA COLLEGES
Senior High School Mathematics Contest
Preliminary Round 1998
Time: 45 minutes
1. The integer 1998 = (n , 1)nn(10n + c) where n and c are positive
integers. It follows that c equals:
(a) 2 (b) 5 (c) 6 (d) 7 (e) 8
2. The value of the sum log 12 + log 23 + log 34 +    + log 109 is:
(a) ,1 (b) 0 (c) 1 (d) 2 (e) 3
3. Four basketballs are placed on the gym oor in the form of a square
with each basketball touching two others. A fth basketball is placed on top
of the other four so that it touches all four of the other balls, as shown. If the
diameter of a basketball is 25 cm, the height, in centimetres, of the centre of
the fth basketball above the gym oor is:

p
(a) 25 2
p
(b) 252 2 (c) 20
p
(d) 252 (1 + 2)
p
(e) 25(1 + 2)
4. Last summer, I planted two trees in my yard. The rst tree came in
a fairly small pot, and the hole that I dug to plant it in lled one wheelbarrow
load of dirt. The second tree came in a pot, the same shape as that of the
rst tree, that was one-and-a-third times as deep as the rst pot and one-
and-a-half times as big around. Let us make the following assumptions:
i) The hole for the second tree was the same shape as for the rst tree.
399

ii) The ratios of the dimensions of the second hole to those of the rst
hole are the same as the ratios of the dimensions of the pots.
Based on these assumptions, the number of wheelbarrows of dirt that I lled
when I dug the hole for the second tree was:
(a) 2 (b) 2:5 (c) 3 (d) 3:5 (e) none of these
5. You have an unlimited supply of 5-gram and 8-gram weights that
may be used in a pan balance. If you use only these weights and place them
only in one pan, the largest number of grams that you cannot weigh is:
(a) 22 (b) 27 (c) 36 (d) 41 (e) there is no largest number of grams
6. If all the whole numbers from 1 to 1;000;000 are printed, the num-
ber of times that the digit 5 appears is:
(a) 100;000 (b) 500;000 (c) 600;000 (d) 1;000;000 (e) 2;000;000
7. The perimeter of a rectangle is x centimetres. If the ratio of two
adjacent sides is a : b, with a > b, then the length of the shorter side, in
centimetres, is:
(a) abx
+b
(b) x2 , b (c) a2+bxb (d) 2(aax+b) (e) 2(abx+b)
8. The sum of the positive solutions to the equation xxpx = (xpx)x
is:
(a) 1 (b) 1 12 (c) 2 41 (d) 2 21 (e) 3 41
9. Two circles, each with a radius of one unit, touch as shown. AB
and CD are tangent to each circle. The area, in square units, of the shaded
region is:
A B

C D
(a)  (b) 4 (c) 2 , 2 (d) 4 ,  (e) none of these
10. A parabola with a vertical axis of symmetry has its vertex at (0; 8)
and an x{intercept of 2. If the parabola goes through (1; a), then a is:
(a) 5 (b) 5:5 (c) 6 (d) 6:5 (e) 7
11. A ve litre container is lled with pure orange juice. Two litres of
juice are removed and the container is lled up with pure water and mixed
400

thoroughly. Then two litres of the mixture are removed and again the con-
tainer is lled up with pure water. The percentage of the nal mixture that
is orange juice is:
(a) 27 (b) 25 (c) 30 (d) 36 (e) 24
12. The lengths of the sides of a triangle are b + 1, 7 , b and 4b , 2.
The number of values of b for which the triangle is isosceles is:
(a) 0 (b) 1 (c) 2 (d) 3 (e) none of these
13. The number of times in one day when the hands of a clock form a
right angle is:
(a) 46 (b) 22 (c) 24 (d) 44 (e) 48
14. In my town some of the animals are really strange. Ten percent of
the dogs think they are cats and ten percent of the cats think they are dogs.
All the other animals are perfectly normal. One day, I tested all the cats and
dogs in the town and found that 20% of them thought that they were cats.
The percentage of the dogs and cats in the town that really are cats is:
(a) 12:5 (b) 18 (c) 20 (d) 22 (e) 22:5
15. A short hallway in a junior high school contains a bank of lockers
numbered one to ten. On the last day of school the lockers are emptied and
the doors are left open. The next day, a malicious math student walks down
the hallway and closes the door of every locker that has an even number.
The following day, the same student again walks down the hallway and, for
every locker whose number is a multiple of three, closes the door if it is open
and opens it if it is closed. On the next day, the student does the same thing
with every locker whose number is divisible by four. If the student continues
this procedure for a total of nine days, the number of lockers that are closed
after the ninth day is:
(a) 4 (b) 5 (c) 6 (d) 7 (e) 8

Last issue we gave the Junior High School Mathematics Contest, Pre-
liminary Round 1998 of the British Columbia Colleges. Here are the \ocial
solutions", which come our way from the organizer, Jim Totten, The Univer-
sity College of the Cariboo.
BRITISH COLUMBIA COLLEGES
Junior High School Mathematics Contest
Preliminary Round 1998
Time: 45 minutes
1. A number is prime if it is greater than one and divisible only by one
and itself. The sum of the prime divisors of 1998 is: (c)
401

Solution. We can factor the number 1998 as follows: 1998 = 2 


999 = 2  32  111 = 2  33  37. Hence, the sum of its prime divisors is
2 + 3 + 37 = 42.
2. Successive discounts of 10% and 20% are equivalent to a single
discount of: (c)
Solution. If P denotes the initial price then the new price after deduct-
ing the two consecutive discounts is P (1 , 0:1)(1 , 0:2) = 0:72P . This gives
the total discount of (1 , 0:72)  100% = 28%.
k
3. Suppose that A = A2 and A 2 B = A , 2B. Then the value of
k k
7 2 3 is: (e)
k k
Solution. According to our de nitions, 7 2 3 = 72 2 32 = 49 2 9 =
49 , 2  9 = 31.
4. The expression that is not equal to the value of the four other ex-
pressions listed is: (d)
Solution.
p The values of the ve expressions p are:
(a) 1 9 + 9 , 8 =p 2, (d) (1 , 9)  (9 , 8) = ,2,
(b) (1 + 9)  p(, 9 + 8) = 2, (e) 19 , 9 , 8 = 2.
(c) ,1  9 + 9 + 8 = 2,
Thus,the expression (d) has a di erent value than the other four expres-
sions.
5. The sum of all of the digits of the number 1075 , 75 is: (c)
Solution. Consider the procedure for subtracting 75 from 1075 \by
hand":
100 : : : 000
,75
99 : : : 925
Hence, the decimal digits of 1075 , 75 are: 5, 2 and seventy-three copies
of 9. The sum of the digits is 5 + 2 + 73  9 = 664.
6. A circle is divided into three equal parts and one part is shaded as in
the accompanying diagram. The ratio of the perimeter of the shaded region,
including the two radii, to the circumference of the circle is: (d)
402

Solution. The ratio is given by


r + r + r r+6r
= 2r6r
+ 6r =  + 3 :
2 2
3
2r = 2r
3
3
7. The value of 1
2 , 2, 12,1 1
2
is: (b)
Solution. We can simplify this compound fraction by working succes-
sively from the bottom to the top of the expression:
1 = 2 , 1 1 = 2 ,1 1 = 2 ,1 1 = 2 ,1 3 = 15 = 45 :
2 , 2, 2,1 1
1
2, 1 2, 2 ( 43 )
2 ( 32 ) 3 4 4

8. If each small square in the accompanying grid is one square cen-


timetre, then the area in square centimetres of the polygon ABCDE is: (a)
Solution. We can nd the area by decomposing the polygon ABCDE
into simpler gures, for example, into three triangles: ABE , BCE , and
CDE.
C D

A E
If we choose AE , CE , and CD as bases of the triangles then the lengths
of the corresponding perpendicular heights are 5, 3, and 8 cm. Hence, the
area of the polygon is 12  4  5 + 12  8  3 + 12  4  8 = 38.
9. A point P is inside a square ABCD whose side length is 16. P is
equidistant from two adjacent vertices, A and B , and the side CD opposite
these vertices. The distance PA equals: (e)
403

Solution. The situation is illustrated by the following diagram, where


x denotes the distance PA.
D C

x
16
P
x x

A E B
The Pythagorean Theorem applied to triangle PEB gives (16,x)2+82 = x2 ,
162 +82
so that 16 , 32x + x + 8 = x , and x = 32 = 10.
2 2 2 2

10. A group of 20 students has an average mass of 86 kg per person.


It is known that 9 people from this group have an average mass of 75 kg
per person. The average mass in kilograms per person of the remaining 11
people is: (b)
Solution. If m ; m2; : : : ; m20 denote the masses of the students then
m1 +m2 ++m20 = 861 . Hence, m1 + m2 +    + m20 = 20  86 = 1720. We
20
can assume, without loss of generality, that the average mass of the rst nine
students is 75; that is, m1 +m29++m9 = 75. Hence, m1 + m2 +    + m9 =
9  75 = 675. The total mass of1045 the remaining 11 people is 1720 , 675 =
1045. This gives the average of 11 = 95.
11. In the following display each letter represents a digit:
3 B C D E 8 G H I
The sum of any three successive digits is 18. The value of H is: (a)
Solution. We have 3 + B + C = 18. Consequently, B + C = 15.
By subtracting this equation from B + C + D = 18 we get D = 3. Now,
D +E +8 = 18 gives E = 10,D = 10,3 = 7. Finally, E +8+G = 18 gives
G = 10 , E = 10 , 7 = 3, and 8 + G + H = 18 gives H = 18 , 8 , 3 = 7.
12. In the accompanying diagram \ADE = 140. The sides are
congruent as indicated. The measure of \EAD is: (e)
D
B
A C E
404

Solution. If \EAD = , then also \ACB = , since triangle ABC


is isosceles. Hence, \CBD = 2 as an external angle of triangle ABC .
Consequently, \ADC = 2 , since triangle BCD is isosceles. Further,
\ECD = \ADC + \CAD as an external angle of triangle ADC . Hence,
\ECD = 2 + = 3 . Now, \AED = \ECD = 3 , because tri-
angle CDE is isosceles. This implies that \CDE = 180 , 6 . Finally,
\ADE = \ADC + \CDE = 2 + 180 , 6 = 180 , 4 . Thus,
180 , 4 = 140. This yields = 10 .
13. The area (in square units) of the triangle bounded by the x{axis
and the lines with equations y = 2x + 4 and y = , 23 x + 4 is: (e)
Solution. Two vertices of the triangle lie on the x{axis, so they are
the x{intercepts of the lines. The x{intercept of the rst line, determined
by the equation 2x + 4 = 0, is ,2. Similarly, the second x{intercept, de-
termined by , 23 x2 + 4 = 0, is 6. Consequently, the length of the base of
the triangle is 6 , (,2) = 8. Since both lines have the same y {intercept
4, they intersect each other and the y{axis at level 4 and, consequently, the
length of the height of the triangle is 4. Therefore, the area of the triangle is
A = 12 (4  8) = 16.
14. Two diagonals of a regular octagon are shown in the accompanying
diagram. The total number of diagonals possible in a regular octagon is: (d)

Solution. Let di , for i = 1; 2; : : : ; 8, denote the number of diagonals


connected to the ith vertex. Then d1 = d2 = : : : = d8 = 5, since each
vertex of the octagon is connected to ve diagonals. On the other hand, each
diagonal joins two vertices. Therefore, in the sum d1 + d2 +    + d8 =
8  5 = 40, each diagonal is counted twice. Hence, the number of diagonals
in the octagon is 20.
15. A local baseball league is running a contest to raise money to send
a team to the provincial championship. To win the contest it is necessary
to determine the number of baseballs stacked in the form of a rectangular
pyramid. The fth and sixth levels from the base of the stack of baseballs are
shown. If the stack contains a total of seven levels, the number of baseballs
in the stack is: (d)
405

Solution. The fth level has 3  4 = 12 balls, the sixth 2  3 = 6


balls, and the seventh 1  2 = 2 balls. We notice that the number of balls
in both sides of the rectangle they form increases by one each time we move
one level down. Thus, the total number of balls is 1  2 + 2  3 + 3  4 +
4  5 + 5  6 + 6  7 + 7  8 = 168.

That completes the Skoliad Corner for this issue. We need suitable
contests and solutions. I welcome any comments, criticisms, or suggestions
for the future direction of this feature.

Advance Announcement
The 1999 Summer Meeting of the Canadian Mathematical Society will
take place at Memorial University in St. John's, Newfoundland, from Satur-
day, 29 May 1999 to Tuesday, 1 June 1999.
The Special Session on Mathematics Education will feature the topic

What Mathematics Competitions do for Mathematics.


The invited speakers are
Ed Barbeau (University of Toronto),
Ron Dunkley (University of Waterloo),
Tony Gardiner (University of Birmingham, UK),
Rita Janes (Newfoundland and Labrador Senior Mathematics League), and
Shannon Sullivan (student, Memorial University).
Requests for further information, or to speak in this session, as well as sug-
gestions for further speakers, should be sent to the session organizers:
Bruce Shawyer and Ed Williams
CMS Summer 1999 Meeting, Education Session
Department of Mathematics and Statistics, Memorial University
St. John's, Newfoundland, Canada A1C 5S7
406

MATHEMATICAL MAYHEM
Mathematical Mayhem began in 1988 as a Mathematical Journal for and by
High School and University Students. It continues, with the same emphasis,
as an integral part of Crux Mathematicorum with Mathematical Mayhem.
All material intended for inclusion in this section should be sent to the
Mayhem Editor, Naoki Sato, Department of Mathematics, Yale University,
PO Box 208283 Yale Station, New Haven, CT 06520{8283 USA. The electronic
address is still
[email protected]
The Assistant Mayhem Editor is Cyrus Hsia (University of Toronto).
The rest of the sta consists of Adrian Chan (Upper Canada College), Jimmy
Chui (Earl Haig Secondary School), Richard Hoshino (University of Waterloo),
David Savitt (Harvard University) and Wai Ling Yee (University of Waterloo).

Shreds and Slices


An Algebraic Relation with a Geometric Twist
Cyrus Hsia
Consider the following algebraic relationship between the positive real num-
bers a, b, and c:
1 = 1 +1:
c a b
If we consider line segments with lengths of a, b, and c, then they are related
to each other as shown in the following diagram.

a  b

 c

The diagram shows the three line segments parallel to each other and
emanating from a common line. This gure and relationship between the
line segments appear a lot. The reader is encouraged to prove this.
407

As a corollary of this fact, here is another geometric example. Let P ,


A, B, and C be points in the plane such that \APC = \CPB = 60 and
A, C , and B are collinear. Show that 1=PC = 1=PA + 1=PB.
P
60 60 

A C B

Another algebraic relation between three positive numbers with an interest-


ing geometric interpretation is the following:
p1c = p1a + p1b :
The reader is encouraged to nd a geometric interpretation for the
above relation before looking at the diagram below. Use a, b, and c as the
length of three line segments, and determine a geometric gure that relates
the three.

a
q q b

qc

It turns out that if a, b, and c are considered to be the lengths of the


radii of three circles, then the circles may all be tangent to a common line
and to each other as shown. Again readers are encouraged to prove this
themselves.
Now what about a generalization? Consider the following algebraic
relation between positive reals a, b, c, and a real number x:
cx = ax + bx:
The rst case then corresponds to the value x = ,1 and the second case to
x = , 12 .
408

The case x = 1 is trivial, as we could interpret it geometrically as a line


segment of length c is made up of the sum of its parts of lengths a and b.
b
c
a
If we wanted to get fancy, we could give the following geometric exam-
ple instead. Consider an equilateral triangle ABC inscribed in a circle, as
shown. P is a point on arc BC . Prove that PA = PB + PC .
A

B C
P
The reader is probably already familiar with the famous case x = 2
known as the Pythagorean Theorem: A triangle with sides a, b, and c is a
right-angled triangle if and only if the lengths satisfy a2 + b2 = c2 .

b c

a
Of course, no discussion about algebraic relations in the above form is
complete without mentioning the notorious Fermat's Last Theorem and the
recent announcement that it has nally been laid to rest. If x is an integer
with x > 2, then the claim is that no solution in the natural numbers exists
for a, b, and c. However, in our general case, the values are real, so we are
not limited by the above result to nding wild and wacky geometric or other
interpretations for it.
409

If the reader is curious, as are we, then try the following exercises to
nd geometric interpretations for special cases of the above relation. The
exercises are explorational and may not have nice solutions, if any. Readers
are welcome to submit any interesting results they nd.
Exercises
1. Let a, b, and c be the lengths of three line segments. Determine how
these three line segments are related geometrically if they satisfy the
relation
(a) c3 = a3 + b3,
(b) pc = pa + b,
p
(c) c12 = a12 + b12 .
2. It is clear that algebraically, all the relations are similar. However,
the geometric interpretations do not appear to be related. Is there a
general geometric description where each of the above geometric gures
is a special case?
3. The algebraic relation clearly does not work for the case x = 0. Is
there a way to de ne the relation so that it would be consistent with
everything else mentioned so far?

We extend congratulations to Ravi Vakil and Alice Staveley, who were


married at St. John's, NF on Monday, 12 October 1998.
410

Mayhem Problems
The Mayhem Problems editors are:
Richard Hoshino Mayhem High School Problems Editor,
Cyrus Hsia Mayhem Advanced Problems Editor,
David Savitt Mayhem Challenge Board Problems Editor.
Note that all correspondence should be sent to the appropriate editor |
see the relevant section. In this issue, you will nd only problems | the
next issue will feature only solutions.
We warmly welcome proposals for problems and solutions. We request
that solutions from this issue be submitted by 1 September 1999, for publi-
cation in issue 8 of 1999.

High School Problems


Editor: Richard Hoshino, 17 Norman Ross Drive, Markham, Ontario,
Canada. L3S 3E8 <[email protected]>
H245. Determine how many distinct integers there are in the set

12  ;  22  ;  32  ; : : : ;  19982  :
1998 1998 1998 1998
H246. Let S(n) denote the sum of the rst n positive integers. We
say that an integer n is fantastic if both n and S (n) are perfect squares. For
example, 49 is fantastic, because 49 = 72 and S (49) = 1+2+3+   +49 =
1225 = 352 are both perfect squares. Find another integer n > 49 that is
fantastic.
H247. Say that the integers a, b, c, d, p, and r form a cyclic set
(a; b; c; d; p; r) if there exists a cyclic quadrilateral with circumradius r, sides
a, b, c, and d, and diagonals p and 2r.
(a) Show that if r < 25, no cyclic set exists.
(b) Find a cyclic set (a; b;c; d; p; r) for r = 25.
H248. Consider a tetrahedral die that has the four integers 1, 2, 3,
and 4 written on its faces. Roll the die 2000 times. For each i, 1  i  4,
let f (i) represent the number of times that i turned up. (So, f (1) + f (2) +
f (3) + f (4) = 2000.) Also, let S denote the total sum of the 2000 rolls.
If S 4 = 6144  f (1)f (2)f (3)f (4), determine the values of f (1), f (2),
f (3), and f (4).
411

Advanced Problems
Editor: Cyrus Hsia, 21 Van Allan Road, Scarborough, Ontario, Canada.
M1G 1C3 <[email protected]>
A221. Construct, using straightedge and compass only, the common
tangents of two non-intersecting circles.
A222. Does there exist a set of n consecutive positive integers such
that for every positive integer k < n, it is possible to pick k of these numbers
whose mean is still in the set?
A223. Proposed by Mohammed Aassila, Universite Louis Pasteur,
Strasbourg, France.
Suppose p is a prime with p  3 (mod 4). Show that for any set of
p , 1 consecutive integers, the set cannot be divided into two subsets so
that the product of the members of the one set is equal to the product of the
members of the other set.
(Generalization of Question 4, IMO 1970)
A224. Proposed by Waldemar Pompe, student, University of War-
saw, Poland.
Let P be an interior point of triangle ABC such that \PBA =
\PCA = (\ABC + \ACB )=3. Prove that
AC = AB :
AB + PC AC + PB

Challenge Board Problems


Editor: David Savitt, Department of Mathematics, Harvard University,
1 Oxford Street, Cambridge, MA, USA 02138 <[email protected]>
C81. Let fan g be the sequence de ned as follows: a0 = 0, a1 = 1,
and an+1 = 4an , an,1 for n = 1, 2, 3, : : : .
(a) Prove that a2n , an,1 an+1 = 1 for all n  1.
1
X

1 
(b) Evaluate arctan 4a2 :
k=1 k
C82. Find the smallest multiple of 1998 which appears as a partial
sum of the increasing sequence
1; 1; 2; 2; 2; 4; 4; 4; 4; 8; : : : ;
in which the number 2k appears k + 2 times (for k a non-negative integer).
412

IMO Report
Adrian Chan, student, UCC, Toronto
This year's Canadian IMO team began with a week of training at the Univer-
sity of Calgary with lavish meal tickets. Then they were o to beautiful and
rocky Kananaskis where we all had an \adventurous" time. Once the team
stepped o the air-conditioned plane and into hot and muggy Taipei, Taiwan,
it marked the team's ocial arrival to the 39th International Mathematical
Olympiad.
The team consisted of the following members: Adrian \Oops I dropped
my..." Birka, Adrian \If You Will" Chan, Jimmy \Nuclear Aerial Strike" Chui,
Mihaela \Baia" Enachescu, Jessie \So Cute" Lei, and Adrian \Nailing Radar"
Tang. Team leader Dr. Christopher \Focus" Small was driven to the edge,
while deputy leader J.P. \It's So Easy" Grossman calmly polished o old
competitions one by one. Special thanks to leader observer Arthur \Rubik's
Cube" Baragar and deputy observer Dorette \Dutch" Pronk for their coaching
and experience. Also, thanks must go to Dr. Bill Sands of the University of
Calgary for organizing such a fun training session.
The contest itself seemed to continue the trend of dicult IMO's and
low medal cuto s. With 76 countries competing, Canada fared extremely
well, bringing back 1 gold, 1 silver, 2 bronze and an honourable mention.
The scores were as follows:
CAN 1 Adrian Birka 10
CAN 2 Adrian Chan 31 Gold Medal
CAN 3 Jimmy Chui 14 Bronze Medal
CAN 4 Mihaela Enachescu 30 Silver Medal
CAN 5 Jessie Lei 13 Honourable Mention
CAN 6 Adrian Tang 15 Bronze Medal
In this year's contest, Canada placed 20th out of 76 countries, up from
last year's 29th ranking. Best of luck to CAN 1, 4, and 6 as they continue
university studies at MIT, Harvard, and Waterloo respectively. CAN 2, 3,
and 5 are all eligible for next year's team. Hopefully there won't be as much
moaning of \Where did I go wrong?" next year around!
Special thanks must also go to Dr. Graham Wright of the Canadian
Mathematical Society for again taking care of the tab, and Professor Ed
Barbeau for his hard work and dedication to the training of potential IMO
candidates through his year-long correspondence program.
Although sometimes things didn't make sense, and the IMO ag some-
how disappeared, the 39th International Mathematical Olympiad ran
smoothly and was de nitely a success. The new experience of a place half-
way around the world with a stimulating culture was new to most of us. Best
of luck to all IMO hopefuls for the 1999 team, as yet another Canadian IMO
journey begins next July in Bucharest, Romania.
413

Bogus Arguments and Arcane Identities


Ravi Vakil
Princeton University

In Euler's time, mathematics was faster and looser than today, and
niceties such as limits were blatantly ignored. Here is an argument of Euler's
that seems to have no right to work, but does nonetheless. We conclude with
some avenues for exploration and an open question.
If r1, : : : , rn are the zeros of a polynomial a0 + a1 x +    + an xn , and
none of the ri are zero (or a0 6= 0), then the negative of the linear term over
the constant term is the sum of the reciprocals of the roots:
, aa10 = r11 +    + r1n :
What about power series? For example, cos x = 1 , x2=2+ x4 =24 ,  ,
so that
cos px = 1 , x + x ,    :
2

2 24
The zeroes of this function are the squares of the odd multiples of =2:
((2n + 1)=2)2, n = 0, 1, 2, : : : . One might hope that the principle for
polynomials given above still holds:
1
1 = X 1
2 n=0 ((2n + 1)=2)2
which can be rewritten as
1
X 1 2 :
= (1)
n=0 (2n + 1) 8
2

This is actually true!


Another possibility is to use sin x, which has power series expansion
sin x = x , x6 + 120 x , :
3 5

Can you use the power series for (sin px)=x to \prove" the identity
X1 1 2 ?
= (2)
n=1 n 6
2

Copyright c 1998 Canadian Mathematical Society


414

Can you relate (2) to (1) by arguing that (2) minus a quarter of (2) is (1)?
P
If you write a short computer program to compute 1000 n =1 n2 , and com-
1

pare it to  2 =6, you'll see that they are indeed very close. In fact, they di er
by almost exactly 1=1000. Can youPexplain why this might be? Can you
guesstimate the di erence between 1000 n=1 (2n+1)2 and  2 =8?
1

And nally, can you conjure up other examples of this sort of argument,
to \prove" other arcane identities? If so, please let us know!
Acknowledgements. This note was inspired by the example of cos px
given in [A].
References
[A] S. Abhyankar, Historical ramblings in algebraic geometry and re-
lated algebra, Amer. Math. Monthly 83 (1976), no. 6, 409{448.
[E] L. Euler, Introductio in Analysin In nitorum, Berlin Academy, 1748.
415

The Fibonacci Sequence


Wai Ling Yee
student, University of Waterloo

The sequence de ned by F0 = 0, F1 = 1, and Fn = Fn,1 + Fn,2 for


n  2 is called the Fibonacci sequence. Named after Leonardo of Pisa, who
is also known as Fibonacci (unsurprisingly), it is one of the most widely stud-
ied sequences of all time. The Fibonacci sequence is an excellent topic with
which to begin learning some basic number theory and various techniques
for working with recurrence relations.
Basic Results
Theorem 1. For all n  1,
Fn2 , Fn,1Fn+1 = (,1)n,1 :
Proof by Induction. When n = 1, F12 , F0 F2 = 12 , 0  1 = 1 =
(,1) , , so the formula holds for n = 1. Assume that the formula holds for
1 1

some n = k, k  1. For n = k + 1,
Fk2+1 , FkFk+2 = Fk2+1 , Fk(Fk+1 + Fk)
= (Fk+1 , Fk )Fk+1 , Fk2
= Fk,1Fk+1 , Fk2
= ,(,1)k,1 by the induction hypothesis
= (,1)k;
so the formula holds for n = k + 1. Therefore, by mathematical induction,
the formula holds for all n  1.
Theorem 2. For all n  1,
F1 + F3 +    + F2n,1 = F2n :
Proof. Using the recurrence relation n times, we have
F2n = F2n,1 + F2n,2
= F2n,1 + F2n,3 + F2n,4
= 
= F2n,1 + F2n,3 +    + F3 + F1 + F0
= F2n,1 + F2n,3 +    + F3 + F1 :
Copyright c 1998 Canadian Mathematical Society
416

Exercise 1. Prove n
X
Fj2 = FnFn+1 :
j =1
Divisibility
Theorem 3. For all m, n  1,
Fn+m = FnFm+1 + Fn,1Fm :
Proof. We will prove this by induction on m. When m = 1,
Fn+1 = Fn  1 + Fn,1  1 = FnF2 + Fn,1F1;
so the formula holds for all n when m = 1. Assume that the formula holds
for all n when m = M . For m = M + 1,
Fn+M +1 = Fn+M + F(n,1)+M
= Fn FM +1 + Fn,1FM + Fn,1FM +1 + Fn,2FM
by the induction hypothesis
= Fn FM +1 + (Fn,1 + Fn,2 )FM + Fn,1 FM +1
= Fn (FM + FM +1 ) + Fn,1 FM +1
= Fn FM +2 + Fn,1FM +1;
so the formula holds for all n for m = M + 1. By mathematical induction,
the formula holds for all m, n  1.
Corollary 4. For all m, n  1, FnjFnm .
Proof. We will prove this by induction on m. When m = 1, Fn certainly
divides itself for every positive integer n. Suppose the statement holds for
all n when m = M . For m = M + 1,
Fn(M +1) = FnM +n = FnM Fn+1 + FnM ,1Fn
by Theorem 3. Since FnM is divisible by Fn by the induction hypothe-
sis, FnM Fn+1 + FnM ,1Fn is also divisible by Fn. This is equivalent to
FnjFn(M +1), so the result holds for m = M + 1. By mathematical induc-
tion, the formula holds for all m, n  1.
Exercise 2. Prove that for every positive integer n, there exist n con-
secutive, composite Fibonacci numbers.
Number Theory 101
We will now de ne a few terms in the interests of formality. For in-
tegers a and b, we say that a divides b if there exists an integer q such that
b = aq, and a is called a divisor of b. Given two non-zero integers a and b,
the largest number which divides both of them, denoted gcd(a; b), is called
417

their greatest common divisor. If gcd(a; b) = 1, then a and b are said to be


relatively prime.
Theorem 5. (The Division Algorithm) Given a positive integer a and
an integer b, there exist unique integers q and r such that b = aq + r and
0  r < a. Then q is called the quotient, and r is called the remainder upon
division of b by a.
Proof. Consider the set
S = fs : s = b , aq  0; q 2 Zg:
S cannot be empty. If b  0, then select q = 0 to give b 2 S. Otherwise,
if b < 0, select q = b so that b , ab = b(1 , a)  0, which means that
b , ab 2 S. Since S is non-empty and contains only non-negative integers,
we can nd the smallest element in S . Call it r.
Suppose r  a. Then 0  r , a = b , aq , a = b , a(q + 1) for some
q, so r , a 2 S and it is smaller than r, contradiction. Thus 0  r < a.
Suppose that we can nd 0  r1 < r2 < a in S and corresponding q1 and
q2. Then b = aq1 + r1 = aq2 + r2, which implies that a(q1 , q2) = r2 , r1.
Thus a divides r2 , r1 . However, we also know that 0 < r2 , r1 < a; that
is, r2 , r1 lies between two consecutive multiples of a and thus cannot be
divisible by a, contradiction. We have shown the existence and uniqueness
of q and r.
Exercise 3. Prove that gcd(a; b) = gcd(a; b , aq ) for any non-zero
integers a and b and any integer q .
Exercise 4. Prove that if a and q are relatively prime, then gcd(a; qb) =
gcd(a; b), where a; b; q are non-zero integers.
Exercise 5. Prove that Fn and Fn+1 are relatively prime.
Number Theory 102
The Euclidean Algorithm. The Euclidean Algorithm is an algorithm used
to determine the greatest common divisor of two numbers. Suppose we
have two distinct positive integers a and b where, without loss of generality,
a < b. By the Division Algorithm, b = aq1 + r1 where 0  r1 < a for unique
integers q1 and r1. If r1 = 0, then a divides b so our greatest common divisor
is a. Otherwise, by Exercise 3, gcd(a; b) = gcd(a; b , aq1 ) = gcd(a; r1) =
gcd(r1; a). In this case, we then repeat the same argument using r1 and a
where we used a and b before, respectively. We have a = r1q2 + r2 where
0  r2 < r1 for unique integers q2 and r2 by the Division Algorithm, and
gcd(r1; a) = gcd(r1; a , q2r1 ) = gcd(r2; r1). Continue applying this argu-
ment. Since the ri s are strictly decreasing and non-negative, there must be
a last remainder, say rn, that is bigger than 0. So we have
418

b = aq1 + r1 ; 0  r1 < a; gcd(a; b) = gcd(r1 ; a);


a = r1 q2 + r2 ; 0  r2 < r1 ; gcd(r1 ; a) = gcd(r2 ; r1 );
r1 = r2 q3 + r3 ; 0  r3 < r2 ; gcd(r2 ; r1 ) = gcd(r3 ; r2 );
 
rn,2 = rn,1 qn + rn ; 0  rn < rn,1 ; gcd(rn,1 ; rn,2 ) = gcd(rn ; rn,1 );
rn,1 = rn qn+1 ; gcd(rn ; rn,1 ) = rn :
We have found that gcd(a; b) = rn .
Theorem 6. For all a, b  1,
gcd(Fa; Fb ) = Fgcd(a;b):
Proof. If a and b are equal, the result is immediate, so assume that
a < b. Apply the Euclidean Algorithm to obtain
b = aq1 + r1; 0  r1 < a;
a = r1q2 + r2; 0  r2 < r1;
r1 = r2q3 + r3; 0  r3 < r2;

rn,2 = rn,1qn + rn; 0  rn < rn,1;
rn,1 = rnqn+1:
We have
gcd(Fa; Fb) = gcd(Fa; Faq1+r1 ) = gcd(Fa ; Faq1,1Fr1 + Faq1 Fr1+1 )
by Theorem 3. Since Faq1 Fr1+1 is a multiple of Fa by Corollary 4,
gcd(Fa ; Fb) = gcd(Fa; Faq1,1 Fr1 + Faq1 Fr1+1 ) = gcd(Fa; Faq1,1 Fr1 )
by Exercise 3. By Exercise 5, gcd(Faq1 ; Faq1,1 ) = 1. Since Fa divides Faq1 ,
gcd(Fa; Faq1 ,1 ) = 1 also. By Exercise 4, since Fa and Faq1,1 are relatively
prime,
gcd(Fa; Faq1 ,1 Fr1 ) = gcd(Fa; Fr1 ) :
We conclude that gcd(Fa; Fb ) = gcd(Fr1 ; Fa). Repeating this argument, we
obtain
gcd(Fr1 ; Fa) = gcd(Fr2 ; Fr1 ) =    = gcd(Frn ; Frn,1 ) :
Since rn divides rn,1 , Frn divides Frn,1 , which implies that gcd(Frn ; Frn,1 )
= Frn . Thus,
gcd(Fa; Fb ) = Frn = Fgcd(a;b) :
Finding Fn Explicitly
The monic quadratic with roots and is
(x , )(x , ) = x2 , ( + )x + :
419

Let and be the roots of x2 , x , 1 in particular. Then, comparing coef-


cients, + = 1 and = ,1. Using this, we can rewrite the recurrence
relation Fn = Fn,1 + Fn,2 as Fn = ( + )Fn,1 , Fn,2 . From this
equation, we obtain
Fn , Fn,1 = (Fn,1 , Fn,2) :
Let sn,1 = Fn , Fn,1 for all n  2. Rewriting the above equation in terms
of the si, we obtain sn,1 = sn,2 . In other words, the sequence fsn g is a
geometric sequence with common ration . We conclude that sn = n,1 s1 .
Similarly, Fn , Fn,1 = (Fn,1 , Fn,2 ), and if we let
tn,1 = Fn , Fn,1 for n  2, then tn = n,1t1 for n  1. Hence,
Fn = , , Fn + , Fn,1 , , Fn,1
= (Fn , Fn,1 ) , ,
(Fn , Fn,1 )
t n , 1 , sn,1
= ,
n,1t1 , n,1 s1
= ,
n,1(F2 , F1 ) , n,1 (F2 , F1)
= ,
n,1(1 , ) , n,1 (1 , )
= , :
Recall that + = 1, so the equation above is, in fact,
n, n
, :
Solving x2 , x , 1 = 0 for the values of and , we have, without loss of
generality, p5 p5
= 21 + and
1 ,
= 2 :
Substituting these values, we conclude that
 p n 
,p5 n
1+ 5
, 1

Fn = 2
p5 2
:
This is called Binet's Formula for the Fibonacci sequence.
Exercise 5 . Let  = (1 +
p5)=2. Prove that F is the integer closest
n n

to p :
5
420

Problems
1. Prove that the product of every four consecutive Fibonacci numbers is
the area of a Pythagorean triangle.
2. Prove that every positive integer can be written as a sum of distinct
Fibonacci numbers.
3. Prove n+1
bX
2 c n , k
Fn = k,1 :
k=1
4. Prove that if Fn is prime and n  5, then n is prime.
5. Prove that Fn + 1 is always composite for n  4.
6. Show that for any positive integer n, among the rst n2 Fibonacci num-
bers, there exists at least one that is divisible by n.
7. De ne a Fibonacci prime to be a Fibonacci number that is prime. Prove
or disprove: There are in nitely many Fibonacci primes. (Note that this
is an open problem.)
421

J.I.R. McKnight Problems Contest 1984


1. Find the real roots of the equation:
x + 3 , 4px , 1 + x + 8 , 6px , 1 = 1 :
q q

(Note: All square roots are to be taken as positive.)


2. Consider a reservoir in the shape of an inverted cone as shown in the
diagram below. Water runs into the reservoir at the constant rate of
2 m3 per minute. How fast is the water level rising when it is 6 metres
deep?

r
 5m -
6

r 6
10 m
6m

? ?

3. Three forces of magnitudes 10 N, 15 N, and 10 N act at angles of 30 ,


70 , and 120 respectively, to the real axis Ox. Using the complex
numbers and the imaginary axis Oy nd the magnitude and direction of
the resultant force.
4. Normals are drawn from the point 154 ; , 34 to the parabola whose
, 

equation is y 2 = 4x. Find the coordinates of the points where the


normals meet the parabola.
5. The horizontal base of a triangular pyramid is an equilateral triangle
QRS, each of whose sides is 20 cm long. The sloping edges of the
pyramid PQRS are respectively 20 cm, 20 cm, and 12 cm long.
(a) Calculate the perpendicular height of the pyramid to the nearest
millimetre.
(b) Calculate the angle of inclination of each of the three edges with
the base to the nearest tenth of a degree.
6. Prove that if tan A = tan3 B and tan 2B = 2 tan C , then A + B , C =
n for some n 2 Z.
422

Swedish Mathematics Olympiad


1988 Qualifying Round
1. Show that the function
f (x) = x , 4px , 1 + 3 + x , 6px , 1 + 8
q q

is constant on the closed interval 5  x  10.


2. Find the rational root of the equation
(2x)log 2 = (3x)log 3 ; x > 0
in the form pq , where p and q are integers.
3. We will call two squares on a chessboard \neighbours" if they have a
side or corner in common. The numbers 1 to 64 are arbitrarily placed
on the 64 squares of a chessboard. Show that there are always two
\neighbours" whose numbers have positive di erence at least 9.
4. A car's tires wear proportionally with the distance driven. Furthermore,
front tires last a km and back tires b km, where a < b. If, after an
appropriate distance is driven, the tires are rotated (that is, back tires
placed on front wheels and front tires on back wheels), the distance
which can be driven without needing to replace any of the tires can be
increased. What is the longest distance which can be driven with a set
of tires, before any new tires must be bought?
5. P , Q, and R are points on the circumference of a circle such that PQR
is an equilateral triangle. S is an arbitrary point on the circumference
of the circle. Consider the lengths of the line segments PS , QS , and
RS. Show that one of them is the sum of the other two.
6. Show that for every positive integer n, there exist positive integers x
and y such that p
x2 + nxy + y2
is an integer.
423

1988 Final Round


1. The sides of a triangle have lengths a > b > c, and the corresponding
perpendiculars have lengths ha , hb , and hc . Show that
a + ha > b + hb > c + hc :
2. Six ducklings swim on the surface of a pond, which is in the shape of a
circle with radius 5 m. Show that, at every instant, two of the ducklings
swim at a distance of at most 5 m from each other.
3. Show that for aribtrary real numbers x1 , x2 , and x3 ,
if x1 + x2 + x3 = 0; then x1 x2 + x2 x3 + x3 x1  0 :
Find all n  4 for which the statement
if x1 + x2 +    + xn = 0; then x1 x2 + x2 x3 +    + xn,1 xn + xn x1  0
is true. (Both sums have n terms.)
4. Let P (x) be a polynomial of degree 3 with exactly three distinct real
roots. Find the number of real roots of the equation
(P 0 (x))2 , 2P (x)P 00 (x) = 0 :
5. Let m and n be positive integers. Show that there exists a constant
> 1, independent of m and n, such that
m < p7 implies that 7 , m2  :
n n2 n2
6. The sequence a1 , a2 , : : : , is de ned by the recursion formula
s
an+1 = a2n + a1 n  1 ;
n
and a1 = 1. Show that one can choose such that
1 an
2  n 2 for all n  1:
424

PROBLEMS
Problem proposals and solutions should be sent to Bruce Shawyer, De-
partment of Mathematics and Statistics, Memorial University of Newfound-
land, St. John's, Newfoundland, Canada. A1C 5S7. Proposals should be ac-
companied by a solution, together with references and other insights which
are likely to be of help to the editor. When a submission is submitted with-
out a solution, the proposer must include sucient information on why a
solution is likely. An asterisk (?) after a number indicates that a problem
was submitted without a solution.
In particular, original problems are solicited. However, other inter-
esting problems may also be acceptable provided that they are not too well
known, and references are given as to their provenance. Ordinarily, if the
originator of a problem can be located, it should not be submitted without
the originator's permission.
To facilitate their consideration, please send your proposals and so-
lutions on signed and separate standard 8 12 "11" or A4 sheets of paper.
These may be typewritten or neatly hand-written, and should be mailed
to the Editor-in-Chief, to arrive no later than 1 April 1999. They may also
be sent by email to [email protected]. (It would be appreciated if
email proposals and solutions were written in LATEX). Graphics les should
be in epic format, or encapsulated postscript. Solutions received after the
above date will also be considered if there is sucient time before the date
of publication. Please note that we do not accept submissions sent by FAX.

2374. [1998: 365] (Correction) Proposed by Toshio Seimiya, Kawasaki,


Japan.
Given triangle ABC with \BAC > 60 . Let M be the mid-point of
BC . Let P be any point in the plane of 4ABC .
Prove that AP + BP + CP  2AM .
2376. Proposed by Albert White, St. Bonaventure University, St.
Bonaventure, NY, USA.
Suppose that ABC is a right-angled triangle with the right angle at C .
Let D be a point on hypotenuse AB , and let M be the mid-point of CD.
Suppose that \AMD = \BMD. Prove that
1. AC 2 MC 2 + 4[ABC ] [BCD] = AC 2 MB 2 ;
2. 4AC 2 MC 2 , AC 2 BD2 = 4[ACD]2 , 4[BCD]2 ,
where [XY Z ] denotes the area of 4XY Z .
(This is a continuation of problem 1812, [1993: 48].)
425

2377. Proposed by Nikolaos Dergiades, Thessaloniki, Greece.


Let ABC be a triangle and P a point inside it. Let BC = a, CA = b,
AB = c, PA = x, PB = y, PC = z, \BPC = , \CPA = and
\APB = .
Prove that ax = by = cz if and only if , A = , B = , C = 3 .
2378. Proposed by David Doster, Choate Rosemary Hall, Wall-
ingford, Connecticut, USA.
Find the exact value of: cot

  , 4 cos  3  :
22 22
2379. Proposed by D.J. Smeenk, Zaltbommel, the Netherlands.
Suppose that M1 , M2 and M3 are the mid-points of the altitudes from
A to BC , from B to CA and from C to AB in 4ABC . Suppose that T1, T2
and T3 are the points where the excircles to 4ABC opposite A, B and C ,
touch BC , CA and AB .
Prove that M1 T1 , M2 T2 and M3 T3 are concurrent.
Determine the point of concurrency.
2380. Proposed by Bill Sands, University of Calgary, Calgary, Al-
berta.
When the price of a certain book in a store is reduced by 1=3 and
rounded to the nearest cent, the cents and dollars are switched. For ex-
ample, if the original price was $43.21, the new price would be $21.43 (this
does not satisfy the \reduced by 1=3" condition, of course). What was the
original price of the book? [For the bene t of readers unfamiliar with North
American currency, there are 100 cents in one dollar.]
2381. Proposed by Angel Dorito, Geld, Ontario.
Solve the equation log2 x = log4 (x + 1).
2382. Proposed by Mohammed Aassila, Universite Louis Pasteur,
Strasbourg, France.
If 4ABC has inradius r and circumradius R, show that
cos2 B ,2 C  2Rr :
 

2383. Proposed by Mohammed Aassila, Universite Louis Pasteur,


Strasbourg, France.
Suppose that three circles, each of radius 1, pass through the same point
in the plane. Let A be the set of points which lie inside at least two of the
circles. What is the least area that A can have?
2384. Proposed by Paul Bracken, CRM, Universite de Montreal,
Quebec.
Prove that 2(3n , 1)n  (3n + 1)n for all n 2 N.
426

2385. Proposed by Joaqun Gomez  Rey, IES Luis Bu~nuel, Alcorcon,


Madrid, Spain.
A die is thrown n  3 consecutive times. Find the probability that the
sum of its n outcomes is greater than or equal to n +6 and less than or equal
to 6n , 6.
2386?. Proposed by Clark Kimberling, University of Evansville,
Evansville, IN, USA.
Write
1 ! 1 ! 3 ! 4 1 ! 6 2 1 ! 8 1 3 2 1 !
1 1 1 3 1 3 4 1 2 3 4 6
(The last ten numbers shown indicate that up to this point, eight 1's,
one 2, three 3's, two 4's and one 6 have been written.)
(a) If this is continued inde nitely, will 5 eventually appear?
(b) Will every positive integer eventually be written?
Note: 11 is a number and not two 1's.
2387. Proposed by Walther Janous, Ursulinengymnasium, Inns-
bruck, Austria.
For xed p 2 N, consider the power sums
n
X
Sp(n) := (2k , 1)p ; where n  1 ;
k=1
so that Sp (n) is a polynomial in n of degree p + 1 with rational coecients.
Prove that
(a) If all coecients of Sp (n) are integers, then p = 2m , 1 for some
m 2 N.
(b)? The only values of p yielding such polynomials are p = 1 and p = 3
(with S1(n) = n2 and S3(n) = 2n4 , n2 ).
427

SOLUTIONS
No problem is ever permanently closed. The editor is always pleased to
consider for publication new solutions or new insights on past problems.

1637. [1991: 114; 1992: 125; 1994: 165] Proposed by George


Tsintsifas, Thessaloniki, Greece.
Prove that X sin B + sin C > 12
A 
where the sum is cyclic over the angles A; B; C (measured in radians) of a
nonobtuse triangle.
Comment by Waldemar Pompe, student, University of Warsaw, Poland.
We can use Crux 2015 [1998: 305] to derive the strengthening of Crux
1637 given on [1994: 165], namely that
X
p
sin B + sin C  9 3
A 
for all triangles ABC .
Without loss of generality, we can assume that A  B  C . Then
1 1 1
A  B  C
and
sin B + sin C  sin A + sin C  sin A + sin B :
By Chebyshev's Inequality and Crux 2015, we have
X sin B + sin C  2 X sin A  X 1   27 3 ;
p
3 A A 
and the result follows.

2257. [1997: 300] Proposed by Waldemar Pompe, student, Univer-


sity of Warsaw, Poland.
The diagonals AC and BD of a convex quadrilateral ABCD intersect
at the point O. Let OK , OL, OM , ON , be the altitudes of triangles 4ABO,
4BCO, 4CDO, 4DAO, respectively.
Prove that if OK = OM and OL = ON , then ABCD is a parallelo-
gram.
428

I. Solution by Con Amore Problem Group, Royal Danish School of Ed-


ucational Studies, Copenhagen, Denmark.
Let OK = OM = h, OA = a, OB = b, OC = c, OD = d, and
\AOB = v . Expressing in two ways the area of 4AOB , we get
1
2
AB  h = 12 ab sin v;
and so
1 = pa2 + b2 , 2ab cos v = 1 1 + 1 , 2  1  1  cos v :
r

h ab sin v sin v b2 a2 b a
Similarly, r
1 = 1 1 + 1 , 2  1  1  cos v ;
h sin v d2 c2 d c
so that
r r
1 + 1 , 2  1  1  cos v = 1 + 1 , 2  1  1  cos v ; (1)
b2 a2 b a d2 c2 d c
and similarly,
r r
1 + 1 + 2  1  1  cos v = 1 + 1 + 2  1  1  cos v : (2)
a2 d2 a d b2 c2 b c
Now, consider another convex quadrilateral A1 B1 C1 D1 , with diagonals in-
tersecting in O1 , and such that \A1 O1 B1 = v , O1A1 = a1 , O1 B1 = 1b ,
O1C1 = 1c , and O1D1 = d1 . The equalities (1) and (2) imply that the oppo-
site sides of A1 B1 C1 D1 are equal in length, which means that A1 B1 C1 D1
is a parallelogram. So a1 = 1c , and 1b = 1d , implying a = c and b = d. This
proves that ABCD is a parallelogram.
II. Solution by the proposer (slightly edited).
If ABCD is a trapezoid with AB k CD, then
AB OK
CD = OM = 1 ;
which means that ABCD is a parallelogram. Hence, assume that ABCD is
not a trapezoid and set P = AB \ CD, Q = AD \ BC . Indeed, P 6= Q. The
assumption on the given quadrilateral says exactly that PO bisects \BPC
and QO bisects \AQB . Thus
AP = AO = AQ ;
PC OC QC
implying that P , O, and Q lie on the Apollonius circle with centre on the
line AC . Similarly, since
BP = BO = BQ ;
PD OD QD
429

P; O, and Q lie on the Apollonius circle with centre on the line BD. This im-
plies that O is the circumcentre of 4POQ; that is, points P and Q coincide,
a contradiction.
Also solved by CHRISTOPHER J. BRADLEY, Clifton College, Bristol, UK;
MICHAEL LAMBROU, University of Crete, Crete, Greece; TOSHIO
SEIMIYA, Kawasaki, Japan; and D.J. SMEENK, Zaltbommel, the Netherlands. There
were also ve incorrect solutions submitted.
Most of the submitted solutions are similar to the proposer's solution.

2260. [1997: 301] Proposed by Vedula N. Murty, Visakhapatnam,


India.
Let n be a positive integer and x > 0. Prove that
n+1
(1 + x)n+1  (n +n1)
n x:
Solution by Florian Herzig, student, Cambridge, UK; Gerry Leversha,
St. Paul's School, London, England; Nick Lord, Tonbridge School, Tonbridge,
Kent, England; and Panos E. Tsaoussoglou, Athens, Greece.
By the AM{GM Inequality applied to the n + 1 positive numbers
n+1
1 1 1 
x +
x; n ; n ; : : : ; n , we have n + 11  nxn , with equality if and only
if x = n1 . This is clearly equivalent to the given inequality.
Also solved by PAUL BRACKEN, CRM, Universite de Montreal, Montreal,
Quebec; CHRISTOPHER J. BRADLEY, Clifton College, Bristol, UK; THEODORE
CHRONIS, student, Aristotle University of Thessaloniki, Greece; KEITH
EKBLAW, Walla Walla, Washington, USA; RUSSELL EULER and JAWAD
SADEK, NW Missouri State University, Maryville, Missouri, USA; RICHARD
I. HESS, Rancho Palos Verdes, California, USA; JOE HOWARD, New Mex-
ico Highlands University, Las Vegas, NM, USA; WALTHER JANOUS, Ur-
sulinengymnasium, Innsbruck, Austria; MICHAEL LAMBROU, University
of Crete, Crete, Greece; PAVLOS MARAGOUDAKIS, Hatzikiriakio, Pireas,
Greece; PHIL McCARTNEY, Northern Kentucky University, Highland Heights,
KY, USA; VICTOR OXMAN, University of Haifa, Haifa, Israel; MICHAEL
PARMENTER, Memorial University of Newfoundland, St. John's, Newfound-

land; HEINZ-JURGEN SEIFFERT, Berlin, Germany; DAVID R. STONE, Georgia
Southern University, Statesboro, Georgia, USA; JOHN VLACHAKIS, Athens, Greece;
EDWARD T.H. WANG, Wilfrid Laurier University, Waterloo, Ontario; ROGER
ZARNOWSKI, TREY SMITH, CHARLES DIMINNIE and GERALD ALLEN (jointly),
Angelo State University, San Angelo, TX, USA; and the proposer. There were also
three incomplete solutions.
The majority of solvers used a standard calculus approach to establish the given
inequality. The only exceptions are the ve listed in the solutions above plus Lambrou
and Maragoudakis, who used Bernoulli's Inequality. Both Janous and Lambrou noted
that the given inequality holds for all positive real n.
430

Janous also generalized the problem by showing that if a > 0, and > > 0
are given real numbers, then the largest constant C = C (a; ; )such that
(a + x)  Cx holds for all x > 0 is given by C = a , .
,
The given problem is the special case when a = = 1 and = n + 1.
By applying the AM{GM Inequality to the n + 1 positive numbers:
x x x , Lord and the proposer obtained (1 + x)n+1  (n + 1)n+1 xn ,
1; n ; n ; : : : ; n
which is stronger than the proposed inequality for x > 1.
nn
Zarnowski et al. commented that when n is odd, the inequality is true for all
real x, while if n is even, there is a number xn  ,2 such that the inequality holds
for all x  xn .

2261. [1997: 301] Proposed by Angel Dorito, Geld, Ontario.


Assuming that the limit exists, nd
2 + N1++::::::
!

Nlim
!1 1 + N + 2+1+::: ;
:::
where every fraction in this expression has the form
a + bc++::::::
b + ac++::::::
for some cyclic permutation a, b, c of 1, 2, N .
[Proposer's comment: this problem was suggested by Problem 4 of
Round 21 of the International Mathematical Talent Search, Mathematics and
Informatics Quarterly, Vol. 6, No. 2, p. 113.]
Solution by Keith Ekblaw, Walla Walla, Washington, USA.
It will be shown that
2 + N1++ 1 +
p5
1 + N + 1+ ,! 2 as N ! 1:
2+

First, consider
1+ 2+ 
JN = N + 2 + NN ++ :
1+ 
Note that
1 + N2++
2 + N1++ > 0:
Thus JN > N and hence JN ! 1 as N ! 1. Now let
2 + N +
 2 + JN
KN = 1 + 2 + 1 :
KN = 1 + N + 1+
1+ = 1 +
JN JN KN
2+
431

Thus as N ! 1 (and hence JN ! 1), KN ! 1 + 1=KN . Letting K =


limN !1 KN , we have K = 1+1=K or K 2 , K , 1 = 0 and so the required
limit is p
K = 1 +2 5 :
Also solved by CHRISTOPHER J. BRADLEY, Clifton College, Bristol, UK;
CON AMORE PROBLEM GROUP, Royal Danish School of Educational Studies,
Copenhagen, Denmark; FLORIAN HERZIG, student, Cambridge, UK; RICHARD
I. HESS, Rancho Palos Verdes, California, USA; WALTHER JANOUS, Ursulinengym-

nasium, Innsbruck, Austria; VACLAV KONECN  Y,
 Ferris State University, Big Rapids,
Michigan, USA; MICHAEL LAMBROU, University of Crete, Crete, Greece; GERRY
LEVERSHA, St. Paul's School, London, England; and the proposer.
Hess calculates that if N is replaced by 3:5, then the expression inside the limit
(which is itself a limit, actually) is equal to 2. Readers may like to nd other \nice"
triples of numbers a; b; c so that the expression
c+
a + bc + a+
+
+ ab+
is rational, say.
The proposer notes (as can be seen from the above proof) that the answer is
still the golden ratio if the 2's in the given expression are replaced by any constant
real number.

2262. [1997: 301] Proposed by Juan-Bosco Romero Marquez, Uni-


versidad de Valladolid, Valladolid, Spain.
Consider two triangles 4ABC and 4A0 B 0 C 0 such that \A  90 and
\A  90 and whose sides satisfy a > b  c and a0 > b0  c0 . Denote the
0
altitudes to sides a and a0 by ha and h0a .
Prove that (a)
1  1 + 1 , (b) 1  1 + 1 .
hah0a bb0 cc0 hah0a bc0 b0c
Solution by Christopher J. Bradley, Clifton College,Bristol,
 UK.
1 1
(a) By the Cauchy-Schwarz inequality on the vectors ; and 0 ; 0 ,
1 1
b c b c
we have 1  1
1 + 1  1 + 1 2 1 + 1 2:


bb0 cc0 b2 c2 b02 c02


1 1 b2 + c2 a2
Now 2 + 2 = 2 2  2 2 , since \A  90 .
b2 c b 2c bc
a a 1
Also 2 2  2 2 2 = 2 , since 12 aha and 12 bc sin A are both
bc b c sin A h a
formulae for the area of 4ABC . Similarly for 4A0 B 0 C 0 .
1
Hence 0 + 0 
1 1 , as required.
bb cc 0 haha
432

 The proof of part (b) is the same, except that it starts with the vectors
1 ; 1  and  1 ; 1 .
b c c0 b0
Also solved by CLAUDIO ARCONCHER, Jundia, Brazil; WALTHER JANOUS,
Ursulinengymnasium, Innsbruck, Austria; MICHAEL LAMBROU, University of Crete,
Crete, Greece; FLORIAN HERZIG, student, Cambridge, UK; RICHARD I. HESS, Ran-
cho Palos Verdes, California, USA; KEE-WAI LAU, Hong Kong; GERRY LEVERSHA,
St. Paul's School, London, England; VICTOR OXMAN, University of Haifa, Haifa,

Israel; HEINZ-JURGEN SEIFFERT, Berlin, Germany; TOSHIO SEIMIYA, Kawasaki,
Japan; GEORGE TSAPAKIDIS, Agrinio, Greece; JOHN VLACHAKIS, Athens, Greece;
and the proposer.
Most of the submitted solutions are similar to the one given above. Several
solvers pointed out that the restrictions on the sides are unnecessary, and that equal-
ity in (a) occurs if and only if \A = \A0 = 90 and b=c = b0 =c0 and in (b) if and
only if \A = \A0 = 90 and b=c = c0=b0 .
Janous proved more generally that for any real number p  1
1  (bb10)p + (cc10)p ; and 1  (bc10)p + (b01c)p .
(ha h0a)p (ha h0a)p

2263. [1997: 364] Proposed by Toshio Seimiya, Kawasaki, Japan.


ABC is a triangle, and the internal bisectors of \B, \C , meet AC , AB
at D, E , respectively. Suppose that \BDE = 30 . Characterize 4ABC .
Solution by the proposer.
Let F be the re ection of E across BD. Since \EBD = \CBD, it
follows that F lies on BC . So \BDF = \BDE = 30 , and DF = DE .
Then 4DEF is equilateral, so EF = ED and \FED = 60. By the Law
of Sines for 4EFC and 4EDC we obtain that
EC = EF = ED = EC ;
sin \EFC sin \ECF sin \ECD sin \EDC
which gives sin \EFC = sin \EDC . It follows that we have either that
\EFC = \EDC , or that \EFC + \EDC = 180 .
Case 1. \EFC = \EDC . Then
\FEC = \DEC = 12 \FED = 30 :
Let I be the intersection of BD and CE . Then
\DIC = \IED + \IDE = 30 + 30 = 60 :
Since \DIC = 90 , 12 \A, we obtain 90 , 21 \A = 60 , so \A = 60 .
Case 2. \EFC + \EDC = 180 . Then
\FED + \FCD = 180 , so that 60 + \FCD = 180.
Thus \FCD = 120 ; that is, \ACB = 120 .
433

Therefore, ABC is a triangle with either \A = 60 or \C = 120.


Also solved by MIGUEL AMENGUAL COVAS, Cala Figuera, Mallorca, Spain;
FRANCISCO BELLOT ROSADO, I.B. Emilio Ferrari, Valladolid, Spain;
CHRISTOPHER J. BRADLEY, Clifton College, Bristol, UK; FLORIAN HERZIG, stu-
dent, Cambridge, UK; WALTHER JANOUS, Ursulinengymnasium, Innsbruck, Austria;
MICHAEL LAMBROU, University of Crete, Crete, Greece; KEE-WAI LAU, Hong Kong;
GERRY LEVERSHA, St. Paul's School, London, England; VICTOR OXMAN, Univer-
sity of Haifa, Haifa, Israel; D.J. SMEENK, Zaltbommel, the Netherlands. There were
also two incomplete solutions submitted.
Herzig and Lambrou have also shown that the characteristic condition is su-
cient, that is, \A = 60 or \C = 120 implies \BDE = 30 .

2265. [1997: 364] Proposed by Waldemar Pompe, student, Univer-


sity of Warsaw, Poland.
Given triangle ABC , let ABX and ACY be two variable triangles
constructed outwardly on sides AB and AC of 4ABC , such that the angles
\XAB and \Y AC are xed and \XBA + \Y CA = 180. Prove that all
the lines XY pass through a common point.
Solution by Toshio Seimiya, Kawasaki, Japan.

Xr Ar rD
,

rP rE r
B
r rC Y

Z
r
We denote the circumcircle of 4ABC by ,. Let BX and CY meet
at Z . Since \XBA + \Y CA = 180 , we get \XBA = 180 , \Y CA
= \ACZ , so that A; B; Z; C are concyclic, that is, Z lies on ,. Let D; E
be the second intersections of AX; AY respectively with ,. Since AX and
AY are xed lines, D and E are xed points. Let P be the intersection
of BE and CD. Since hexagon ADCZBE is inscribed in ,, by Pascal's
Theorem the intersections of AD and BZ , of DC and BE , and of CZ and
EA are collinear. Therefore variable line XY always passes through the
xed point P . [Editorial note: if the diagram di ers from the one shown,
for example if Z lies between X and B , the proof still works with minor
changes.]
Also solved by CHRISTOPHER J. BRADLEY, Clifton College, Bristol, UK;
FLORIAN HERZIG, student, Cambridge, UK; MICHAEL LAMBROU, University of
Crete, Crete, Greece; MAR I A ASCENSION
 LOPEZ
 CHAMORRO, I.B. Leopoldo Cano,
434

Valladolid, Spain; D.J. SMEENK, Zaltbommel, the Netherlands; and the proposer.
One incorrect solution and one comment were sent in.
Seimiya and the proposer had similar solutions.
Herzig notes that, if the xed angles are chosen to be AXB and AY C instead,
then the lines XY still pass through a xed point. Readers may like to show this
themselves.

2266. [1997: 364] Proposed by Waldemar Pompe, student, Univer-


sity of Warsaw, Poland.
BCLK is the square constructed outwardly on side BC of an acute
triangle ABC . Let CD be the altitude of 4ABC (with D on AB ), and let
H be the orthocentre of 4ABC . If the lines AK and CD meet at P , show
that
HP = AB :
PD CD
I. Solution by Florian Herzig, student, Cambridge, UK.
M

N
K B
Y
D
X
P A
H

L C
Construct a square ABMN outwardly on 4ABC . Let X and Y be
the points of intersection of AK; BC and AB; CM respectively. The ro-
tation through a right angle about B maps 4MBC onto 4ABK . Hence
AK and CM are perpendicular and it follows that AP and CD are alti-
tudes in 4AY C . Therefore P is the orthocentre in that triangle, and as a
consequence Y P ? AC or Y P k BH . Thus
HP = BY = BM = AB :
PD Y D CD CD
435

II. Solution by Toshio Seimiya, Kawasaki, Japan.


K B
D
S P
H A

L C
Let S be a point on AK such that DS ? BC and so DS k AH k BK .
Since AH k DS we get
HP = AH : (1)
PD DS
Since DS k BK and BK = BC , we have
AD = DS = DS : (2)
AB BK BC
Since AH ? BC and CD ? AB we get \HAD = \BCD. Moreover we
have \HDA = \BDC (= 90 ), so that 4HAD  4BCD. Thus
AH = AD : (3)
BC CD
From (2) and (3) we have
AH = AH  BC = AD  AB = AB ;
DS BC DS CD AD CD
so that we obtain from (1) that
HP = AB :
PD CD
Also solved by FRANCISCO BELLOT ROSADO, I.B. Emilio Ferrari, Valladolid,
Spain; CHRISTOPHER J. BRADLEY, Clifton College, Bristol, UK; WALTHER JANOUS,
Ursulinengymnasium, Innsbruck, Austria; VACLAV  KONECN Y,
 Ferris State Univer-
sity, Big Rapids, Michigan, USA; MICHAEL LAMBROU, University of Crete, Crete,
Greece; GERRY LEVERSHA, St. Paul's School, London, England; D.J. SMEENK, Zalt-
bommel, the Netherlands; JOHN VLACHAKIS, Athens, Greece; and the proposer.
The proposer's solution was the same as Herzig's. Most other solvers used
either similar triangles (as in II), trigonometry, or coordinates.
436

2268. [1997: 364] Proposed by Juan-Bosco Romero Marquez, Uni-


versidad de Valladolid, Valladolid, Spain.
Let x, y be real. Find all solutions of the equation
s
2xy + x2 + y 2 = pxy + x + y :
x+y 2 2
Solution s
by Nikolaos Dergiades, Thessaloniki, Greece.
Let A =
x2 + y2 and B = pxy. Then
2
2(A2 + B2) = (x + y)2 and 2(A2 , B 2) = (x , y )2
and the given equation yields
A , B = x +2 y , x2+xyy
or
2(A2 , B 2) = (x , y )2
A+B x+y
(x , y ) = (x , y)2 :
2

A+B x+y
Therefore, we have either (x , y )2 = 0 (which implies that x = y ) or
A + B = x + y. Let us consider A + B = x + y:
A + B = x + y () (A + B)2 = 2(A2 + B2)
() (sA , B)2 = 0 () A = B
() x2 + y2 = pxy
2
() (x , y)2 = 0 () x = y
In conclusion, all solutions have x = y 6= 0, because x + y 6= 0.
Also solved by HAYO AHLBURG, Benidorm, Spain; PAUL BRACKEN,
CRM, Universite de Montreal; CHRISTOPHER J. BRADLEY, Clifton Col-
lege, Bristol, UK; CON AMORE PROBLEM GROUP, Royal Danish School
of Educational Studies, Copenhagen, Denmark (2 solutions); CHARLES R.
DIMINNIE, Angelo State University, San Angelo, TX, USA; DAVID DOSTER,
Choate Rosemary Hall, Wallingford, Connecticut, USA; RUSSELL EULER and
JAWAD SADEK, NW Missouri State University, Maryville, Missouri, USA;
WALTHER JANOUS, Ursulinengymnasium, Innsbruck, Austria; MICHAEL
LAMBROU, University of Crete, Crete, Greece; KEE-WAI LAU, Hong
Kong; VICTOR OXMAN, University of Haifa, Haifa, Israel; HEINZ-JURGEN 
SEIFFERT, Berlin, Germany; D.J. SMEENK, Zaltbommel, the Netherlands; DAVID
R. STONE, Georgia Southern University, Statesboro, Georgia, USA; ROGER
437

ZARNOWSKI, Angelo State University, San Angelo, Texas, USA; and the proposer.
There were 14 incorrect solutions submitted, 11 of which simply did NOT exclude
the origin from the solution set.

2270. [1997: 365] Proposed by D.J. Smeenk, Zaltbommel, the Neth-


erlands.
Given 4ABC with sides a, b, c, a circle, centre P and radius  inter-
sects sides BC , CA, AB in A1 and A2 , B1 and B2 , C1 and C2 respectively,
so that
A1A2 = B1B2 = C1C2 =   0:
a b c
Determine the locus of P .
Solution by the proposer.
[Assume that no two sides of the triangle are equal.] The distance from
s

P to BC is x = 2 ,  4a ;
2 2

P to CA is y = 2 ,  4b ;
2 2

P to AB is z = 2 ,  4c :
2 2

x2 , y2 b2 , a2
It follows that 2 2 = 2 2 , or
y ,z c ,b
x2(c2 , b2) + y2(a2 , c2) + z2(b2 , a2) = 0 : (1)
Considering x; y;z to be the triangular coordinates of P with respect to
4ABC , we conclude that (1) represents a conic K . Note that K passes
through the incentre I (1; 1; 1) and the excentres Ia (,1; 1; 1); Ib (1; ,1; 1),
and Ic (1; 1; ,1), [and also the circumcentre O(cos A; cos B; cos C )]. So K
is the conic through O of the pencil determined by I; Ia ; Ib ; Ic . Since the
degenerate conics of the pencil are degenerate orthogonal hyperbolas (that
is, pairs of perpendicular lines), K must be an orthogonal hyperbola.
Also solved by CHRISTOPHER J. BRADLEY, Clifton College, Bristol, UK.
Neither solver mentioned the obvious special cases:
The locus is a pair of perpendicular lines when 4ABC is isosceles, and just
the four points I; Ia ; Ib ; Ic when equilateral.
Bradley points out that it is clear from the statement of the problem (with no
need for coordinates) that the locus includes I; Ia ; Ib ; Ic (when  = 0) and O (when
 = 1).
438

2271. [1997: 365] Proposed by F.R. Baudert, Waterkloof Ridge,


South Africa.
A municipality charges householders per month for electricity used ac-
cording to the following scale:
rst 400 units | 4.5cj per unit;
next 1100 units | 6.1cj per unit;
thereafter | 5.9cj per unit.
If E is the total amount owing (in dollars) for n units of electricity used,
nd a closed form expression, E (n).
Solution by Michael Lambrou, University of Crete, Greece.
We may view the charges as consisting of
(i) 4.5cj per unit and, additionally,
(ii) a surcharge of 6:1 , 4:5 = 1:6cj per unit, but with
(iii) a refund of 6:1 , 5:9 = 0:2cj per unit for units consumed in excess of
400 + 1100 = 1500.
So (in dollars) the amount owing for n units is:
45 16 2
1000 n + 1000 maxf0; n , 400g , 1000 maxf0; n , 1500g:
Now writing maxfa; bg = 21 (ja , bj + a + b), this simpli es to:
1 f52n + 8jn , 400j , jn , 1500j , 1700g :
1000
Also solved by CHRISTOPHER J. BRADLEY, Clifton College, Bristol, UK;
THEODORE CHRONIS, student, Aristotle University of Thessaloniki, Greece; HANS
ENGELHAUPT, Franz{Ludwig{Gymnasium, Bamberg, Germany; FLORIAN HERZIG,
student, Cambridge, UK; WALTHER JANOUS, Ursulinengymnasium, Innsbruck, Aus-

tria; VACLAV KONECN Y,
 Ferris State University, Big Rapids, Michigan, USA; KEE-
WAI LAU, Hong Kong; and the proposer. There were two incorrect solutions submit-
ted. Some solvers used unit step functions instead of absolute value to express the
answer.

2272?. [1997: 365] Proposer unknown (please identify yourself!)


Write r - s if there is an integer k satisfying r < k < s. Find, as a
function of n (n  2), the least positive integer satisfying
k - k - k -    - k - k:
n n,1 n,2 2
439

Solution by Florian Herzig, student, Cambridge, UK (modi ed by the


editor).
Let kn denote the least positive integer k satisfying
k - k - k -   k - k (1)
n n,1 n,2 2
$ %
We claim that kn =
n + x n
2
+ 1 , where xn = bn + 1 , 2 n , 1c.
p
2 p 
We rst show that kn satis es (1). To this end, letpu = kn . Then for all
integers a = 1; 2;    ; u , 1 we have, from a  kn , 1 that a(a + 1) 
pk (pk , 1) < k . Hence kn , kn > 1, which implies kn - kn .
n n n a a+1 a+1 a
Therefore we have
kn - kn -    - kn  k : (2)
u u,1 2 n
Next we show that for all integers a = u + 1; u + 2;    ; n
n + xn , a < kan < n + xn , a + 1: (3)
In fact, the left inequality in (3) holds for all a = 1; 2;    ; n. To see this, note
that from kn >

n + x n
2
we get a2 , axn , an + kn = a ,
n + xn 2 +
2 2
kn , 2

n + x n
2
> 0, and thus n + xn , a < a : k n

On the other hand, note that the right inequality in (3) is equivalent to

a, n + x n + 1 2
<

n + x n + 1 2
, kn: (4)
2 2
Since kn >

n + xn 2, we have
2
kn + 1  n +2 xn + 1  n + x2n + 1 :
jpk  
a u+1=
[Ed: The last inequality holds since n + xn is an integer.]
Hence it suces to establish (4) for a = n.
Substituting a = n into the right inequality of (3), we need to show that
kn < nxn + n. From n , 2pn , 1 < xn < n we get (n, xn )2 < 4(n , 1)
or (n + xn )2 + 4 < 4(nxn + n). Hence kn 

n + xn 2 + 1 < nx + n.
2 n
Therefore (3) holds, and by setting a = u + 1; u + 2;    ; n we get
kn < xn + 1 < kn <    < xn + n , u , 1 < kn < xn + n , u < kn :
n n,1 u+1 u
440

Hence
kn - kn    - kn - kn : (5)
n n,1 u+1 u
From (2) and (5) we conclude that kn satis es (1).
 if k is any integer satisfying (1), then k  kn . To
Now we show that
k
this end, let x =
n .We rst show that x  xn. Since there exists an
k
integer z such that < z <
k and x = k < z, we have z , x  1
n n,1 n
k k k
and hence
n , 1 > x + 1. Similarly, n , 2 > x + 2, n , 3 > x + 3,    ,
k > x + n , 1. That is, for all a = 1; 2;    ; n we have k > (n , a)(x + a)
1
or k  (n , a)(x + a) + 1 = ,a2 + a(n , x) + nx + 1: (6)
Hence
k  n ,2 x + nx + 1 , a , n ,2 x :
 2  2
(7)
k
Note that x  1. [Ed: If k  n , 1, then 0 < <
k  1, contradicting
n n,1
k - k . Hence k  n].
n n,1
On the other hand, it is clear that xn  n , 1. Suppose, contrary to
what we claim, that x < xn . Then we have 1  x < xn  n , 1 and so
2  n , x < n or 1  n ,2 x < n2 . (Here we must assume that n  3.
The case when n = 2 can be treated
 separately, since it is easy to verify that
n , x 
k2 = 3.) Hence we may let a = 2 in (6) and (7) and obtain
n
k 2 ,
x 2
+ nx + 1 ,

n , x  
n , x 2

2 , 2 : (8)
Since the right side of (7) is an integer and since the last squared term in (8)
1
is either 0 or we get
4
$ % $ %
k n , x 2 + nx + 1 = n + x 2 + 1 : (9)
2 2
$ $ %% $ %
Thus x =
k  1

n + x 2 + 1 = 1  n + x 2 + 1 .
n n 2 n 2 n
441

bz c 

z
 
[Ed: It is known and easy to show that
n = n for all real
numbers z and positive integers n.]
$  % $  %
Hence 0 
1 n + x 2
1 1 n , x 2
1
n 2 !+ n , x = n 2 + n , which
implies that
1  n , x 2 + 1  1 or  n , x 2  n , 1.
n 2 2
p p
Thus n , x < 2 n , 1 or x > n , 2 n , 1, from which we get x 
bn +1 , 2pn , 1c = xn , a contradiction. Hence x  xn . Therefore we may
$ x by
replace xn % in (6), (7), and (9) and conclude that
k n + x 2
n + 1 = k . This completes the proof.
2 n
Also solved by PETER TINGLEY, student, University of Waterloo, Waterloo,
Ontario. There was one incorrect solution. Tingley gave the answer
$ pn , 1 + 1c 2 %
p
kn = nbn , 2 n , 1 + 1c + n , bn , 2 +1
2
which is readily seen to be the same as the one obtained by Herzig. The proposer
had conjectured that

kn = 11 + (n , m)2 if m2  n , 2
+ (n , m)2 + (n , m) otherwise
 p 
where m = 1 + 24n , 7 and had veri ed it for 2  n  600 using a computer.
In a private communication Tingley has actually proved that this conjectured formula
is equivalent to the answer given by Herzig and himself. Interested readers may nd
the proof of this fact quite challenging.

2273. [1997: 366] Proposed by Tim Cross, King Edward's School,


Birmingham, England.
Consider the sequence of positive integers: f1, 12, 123, 1 234, 12 345,
: : : g, where the next term is constructed by lengthening the previous term at
its right-hand end by appending the next positive integer. Note that this next
integer occupies only one place, with \carrying" occurring as in addition: thus
the ninth and tenth terms of the sequence are 123 456 789 and 1 234 567 900
respectively.
Determine which terms of the sequence are divisible by 7.
Solution by Heinz-Jurgen Sei ert, Berlin, Germany.
The sequence fan g under consideration satis es the recurrence
a1 = 1 and an = 10an,1 + n for n  2:
442

A simple induction argument shows that


81an = 10n+1 , 9n , 10; n 2 N:
Let n 2 N. Applying the Euclidean Algorithm twice, we see that there ex-
ist non-negative integers j; k; r such that 0  k  6, 0  r  5, and
n = 42j + 6k + r. Since 342j  36k  1 (mod 7) by Fermat's Little
Theorem, it follows that
4an  3r+1 + 2k , 2r , 3 (mod 7):
The following table gives the remainder when the expression on the right
hand side of the above congruence is divided by 7:
r nk 0 1 2 3 4 5 6
0 0 2 4 6 1 3 5
1 4 6 1 3 5 0 2
2 6 1 3 5 0 2 4
3 2 4 6 1 3 5 0
4 1 3 5 0 2 4 6
5 2 4 6 1 3 5 0
Inspecting this table and using the above congruence, we see that an is
divisible by 7 if and only if n  0; 22; 26; 31; 39; or 41 (mod 42).
Also solved by CHRISTOPHER J. BRADLEY, Clifton College, Bristol, UK;
MIGUEL ANGEL CABEZON  OCHOA, Logro~no, Spain; HANS ENGELHAUPT, Franz{
Ludwig{Gymnasium, Bamberg, Germany; FLORIAN HERZIG, student, Cambridge,
UK; RICHARD I. HESS, Rancho Palos Verdes, California, USA; WALTHER JANOUS,
Ursulinengymnasium, Innsbruck, Austria; MICHAEL LAMBROU, University of Crete,
Crete, Greece; KEE-WAI LAU, Hong Kong; GERRY LEVERSHA, St. Paul's
School, London, England; J.A. MCCALLUM, Medicine Hat, Alberta; JOEL
SCHLOSBERG, student, Robert Louis Stevenson School, New York, NY, USA; TREY
SMITH, GERALD ALLEN, NOEL EVANS, CHARLES DIMINNIE, AND ROGER
ZARNOWSKI (jointly), Angelo State University, San Angelo, Texas; DAVID R. STONE,
Georgia Southern University, Statesboro, Georgia, USA; PAUL YIU, Florida Atlantic
University, Boca Raton, Florida, USA; and the proposer. There was one incorrect
solution submitted.

2274. [1997: 366] Proposed by Vaclav Konecny, Ferris State Uni-


versity, Big Rapids, Michigan, USA.
n Y
X m
A. Let m be a non-negative integer. Find a closed form for (k + j ).
k=1 j =0
n Y
X m
B. Let m 2 f1; 2; 3; 4g. Find a closed form for (k + j )2.
k=1 j =0
443

C?. Let m and j (j = 0; 1; : : : ; m) be non-negative integers. Prove or


n Y
X m mY
+1
disprove that (k + j ) j is divisible by (n + j ).
k=1 j =0 j =0
I. Solution by Florian Herzig, student, Cambridge, UK.
A. Note that
n Y
X m n
X ( m + k )! Xn m + k
(k + j ) = (k , 1)! = (m + 1)! k=1 k , 1
k=1 j =0 k =1

m + 1  
m + 2  
m + 3  
m + n 
= (m + 1)! 0 + 1 + 2 + + n, 1 :
This combinatorial sum [inside the square brackets] is well-known
,m+3 and can
be evaluated as follows. Note that the
,m+4
rst two terms add to 1
which in
turn adds with the third term to 2 , and so on until we obtain the desired
closed form mn+,n1+1 in the end. Thus
, 

n Y
m
(k + j ) = (m + 1)! m n+,n 1+ 1
 
X

k=1 j =0
= (m(m++2)(nn+,1)!1)! = n(n + 1) :m: :+(n2+ m + 1) :

B. The expressions reduce to sums of the form nk=1 km . With the help
P

of a calculator I got
!
n + 2 2(3n2 + 6n + 1) for m = 1;
3 5
!
n + 3 12(2n + 3)(5n2 + 15n + 1) for m = 2;
4 35
!
n + 4 8(35n4 + 280n3 + 685n2 + 500n + 12) for m = 3;
5 21
!
n + 5 40(126n5 + 1575n4 + 6860n3 + 12075n2 + 7024n + 60) for m = 4:
6 77

C. I assume that \is divisible by" means polynomial division.


Clearly the j should be positive integers, and in this case I will prove
that the claim is true. De ne
n Y
X m
P (n) = (k + j ) j ;
k=1 j =0
444

a polynomial of degree ( nj=1 j ) + 1 (since each nk=1 kl is a polynomial


P P

of degree l + 1). Then


n
X m
Y m+X
n+1 Y
m
P (n) = (k + j ) j = (k , m , 1 + j ) j ;
k=,m j =0 k=1 j =0
as in the rst of these sums all the terms for k  0 vanish. Hence, for all
integers a such that ,m  a  0 we get
m+X
a+1
P (a) = (k , m , 1) 0 (k , m) 1 : : : (k , 1) m = 0;
k=1
since each term is zero. [Thus n , a must be a factor of P (n) for each such
a, so P (n) must be divisible by each of n; n + 1; : : : ; n + m. | Ed.] For
n = ,m , 1 the above sum is empty and so PP(,mm +n+1
, 1) = 0 as well (to
avoid empty sums one can instead write P (n) as k=0 pQ (k) , p(0) where
p(k) is the above product). Therefore P (n) is divisible by mj=0+1(n + j ) as
claimed.
II. Solution to part A by Michael Lambrou, University of Crete, Greece.
A. From the identity
2 3
m mY mY
(k + j ) = m 1+ 2 4 (k + j ) , (k , 1 + j )5
Y +1 +1

j =0 j =0 j =0
(easily veri ed by considering the common factor m j =0 (k + j ) of the two
Q

products on the right), we obtain telescopically


2 3
n Y
m mY mY mY
(k + j ) = m 1+ 2 4 (n + j ) , j 5 = m 1+ 2 (n + j ):
X +1 +1 +1

k=1 j =0 j =0 j =0 j =0
[Editorial note: Lambrou also solved parts B and C.]
All three parts also solved by G. P. HENDERSON, Garden Hill, Ontario. Parts A
and B only solved by THEODORE N. CHRONIS, Athens, Greece; WALTHER JANOUS,
Ursulinengymnasium, Innsbruck, Austria; and the proposer.
As Herzig mentions above, part A at least is a fairly familiar result. (For ex-
ample, see formula 2.50, page 50 of Concrete Mathematics by Graham, Knuth and
Patashnik.) In fact it was proposed for publication in CRUX in 1991 by an Edmonton
high school student, Jason Colwell, but was not accepted by the then editor.
Chronis notes that, in the solution for part B, when m = 4, the fth degree
polynomial has 2n + 5 as a factor.
445

2275. [1997: 366] Proposed by M. Perisastry, Vizianagaram, Andhra


Pradesh, India.
Let b > 0 and ba  ba for all a > 0. Prove that b = e.
I. Solution by Gerald Allen, Charles Diminnie, Trey Smith and Roger
Zarnowski (jointly), Angelo State University, San Angelo, TX, USA; Russell
Euler and Jawad Sadek (jointly), NW Missouri State University, Maryville,
Missouri, USA; Michael Parmenter, Memorial University of Newfoundland,
St. John's, Newfoundland; Reza Shahidi, student, University of Waterloo,
Waterloo, Ontario; George Tsapakidis, Agrinio, Greece; and John Vlachakis,
Athens, Greece.
Let f (x) = bx , bx for x > 0. Since f (x)  0 for all x > 0 and
f (1) = 0, it follows that f has a relative (as well as an absolute) minimum
at x = 1.
Since f 0 (1) exists, we have f 0 (1) = 0; that is, b ln b , b = 0. Since
b > 0, we get ln b = 1 or b = e.
II. Solution by Theodore Chronis, student, Aristotle University of
Thessaloniki, Greece; Florian Herzig, student, Cambridge, UK; Michael
Lambrou, University of Crete, Crete, Greece; Gerry Leversha, St. Paul's
School, London, England; Vedula N. Murty, Visakhapatnam, India;
Heinz-Jurgen Sei ert, Berlin, Germany; and David R. Stone, Georgia South-
ern University, Statesboro, Georgia, USA.
The given inequality is equivalent to ba,1  a for all a >0n. Letting
a = 1 + 1 where n 2 N, we get b n1  1 + 1 , or b  1 + 1 . Hence
n n n
b  nlim 1 + 1 n = e.
!1 n
On the other hand, letting a = 1 +

1 ,1 = n , where n 2 N,
n n + 1 n+1
1 1
we get from b1,a  , that b n+1  1 + n , or b  1 + n1
1 . Hence
a
b  nlim 1

+ 1 n = e. Therefore, b = e.
!1 n
Also solved by FRANK P. BATTLES, Massachusetts Maritime Academy, Buz-
zards Bay, MA, USA; CHRISTOPHER J. BRADLEY, Clifton College, Bristol, UK; CON
AMORE PROBLEM GROUP, Royal Danish School of Educational Studies, Copen-
hagen, Denmark; LUZ M. DeALBA, Drake University, Des Moines, IA, USA; DAVID
DOSTER, Choate Rosemary Hall, Wallingford, Connecticut, USA; KEITH EKBLAW,
Walla Walla, Washington, USA; HANS ENGELHAUPT, Franz{Ludwig{Gymnasium,
Bamberg, Germany; F.J. FLANIGAN, San Jose State University, San Jose, Califor-
nia, USA; RICHARD I. HESS, Rancho Palos Verdes, California, USA; JOE HOWARD,
New Mexico Highlands University, Las Vegas, NM, USA; WALTHER JANOUS, Ur-

sulinengymnasium, Innsbruck, Austria; VACLAV KONECN Y, Ferris State University,
Big Rapids, Michigan, USA; KEE-WAI LAU, Hong Kong; D.J. SMEENK, Zaltbommel,
the Netherlands; DIGBY SMITH, Mount Royal College, Calgary, Alberta; and the
446

proposer. There was one incorrect solution.


Although the problem did not ask to show that the condition b = e is both
necessary and sucient, a few solvers did provide a proof of the simple fact that
ex  ex for all x > 0.

2277. [1997: 431] Proposed by Joaqun Gomez


 Rey, IES Luis Bu~nuel,
Alcorcon, Madrid, Spain.
For n  1, de ne
un = (11;n) ; (22;n) ; : : : ; (nn,,1;1n) ; (n;nn) ;
 

where the square brackets [ ] and the parentheses ( ) denote the least com-
mon multiple and greatest common divisor respectively.
For what values of n does the identity un = (n , 1)un,1 hold?
Solution by Florian Herzig, student, Cambridge, UK.
We rst introduce some notation:
for any prime p, let (n)p = maxf jp  ng, [n]p = maxf jp divides ng.
Thus p[n]p jjn. For each k = 1; 2;    ; n we determine a = a(k) such that
pajjk. If a  [n]p, then since pajn, we have pajj(k; n) and so p 6 j k . (k; n)
If a > [n]p , then since p[n]p jk we have p[n]p jj(k; n) and so pa,[n]p jj
k .
(k; n)
Thus the highest power of p in any
k
Y (k;n) arises when a = (n)p. This shows
that un = p(n)p ,[n]p , where the product is over all primes. Note that
Y
p
n,1= p[n,1]p and hence un = (n , 1)un,1 is equivalent to
p
(n)p , [n]p = (n , 1)p for all primes p (1)
We distinguish two cases:
Case (i) Suppose n is a prime power, say n = q b where q is a prime and
b > 0. For p 6= q, (1)b is satis ed since (n)p = (n , 1)p and [n]p = 0. For
p = q we have n = p and so (n)p = [n]p = b and (n , 1)p = b , 1. Hence
(1) holds if and only if b , 1 = 0; that is, b = 1.
Case (ii) If n is not a prime power, then (n)p = (n , 1)p for all primes
p. Hence (i) holds if and only if [n]p = 0 for all primes p, and so n = 1.
[Ed: Clearly n = 1 is not a solution, since u0 is unde ned.]
Therefore un = (n , 1)un,1 if and only if n is a prime.
447

Also solved by ED BARBEAU, University of Toronto, Toronto; NIKOLAOS


DERGIADES, Thessaloniki, Greece; WALTHER JANOUS, Ursulinengymnasium, Inns-

bruck, Austria; VACLAV KONECN Y, Ferris State University, Big Rapids, Michigan,
USA; JOEL SCHLOSBERG, student, Robert Louis Stevenson School, New York, NY,
USA; MICHAEL LAMBROU, University of Crete, Crete, Greece; GERRY LEVERSHA,
St. Paul's School, London, England; DAVID R. STONE, Georgia Southern University,
Statesboro, Georgia, USA; and the proposer.
From the proof given above, it is not dicult to see that, in fact, we have
un = [1; 2;    ; n]=n. This was explicitly pointed out by Kone c ny , Lambrou, and
the proposer, but only Lambrou and the proposer actually gave a proof.

2278. [1997: 431] Proposed by Joaqun Gomez  Rey, IES Luis Bu~nuel,
Alcorcon, Madrid, Spain.
Determine the value of an , which is the number of ordered n{tuples
(k2 ; k3; : : : ; kn ; kn+1) of non-negative integers such that
2k2 + 3k3 + : : : + nkn + (n + 1)kn+1 = n + 1:
I. Solution by Michael Lambrou, University of Crete, Crete, Greece.
We show that an = p(n + 1) , p(n) for n  1 where p(m) denotes
the number of partitions of m into positive integral parts. Our argument is
based on the well-known observation that to a partition of m where lk k's
appear (k = 1; 2; : : : ; m), so that
1l1 + 2l2 +    + mlm = m ; (1)
corresponds the ordered m-tuple (l1 ; l2 ; : : : ; lm ). Conversely, to any given
ordered m-tuple (l1 ; l2 ; : : : ; lm ) of positive integers satisfying (1), there cor-
responds a partition of m.
For xed n  1 consider the partitions of n + 1 as above. They are of
two types:
(a) those for which the number 1 is absent in the decomposition; or
(b) those for which the number 1 appears at least once.
The number of partitions of type (a) is clearly an . Moreover, for each
partition in case (b), if we delete one 1, we get a partition of n. Conversely,
every partition of n with an extra 1 added on gives a partition of n + 1 of
type (b). Clearly then p(n + 1) = an + p(n), as required.
II. Solution by Walther Janous, Ursulinengymnasium, Innsbruck, Aus-
tria.
We deal more generally with the equation:
k1 + 2k2 +    + (j , 1)kj,1 + (j + 1)kj+1 +    + (n + 1)kn+1 = n + 1
where j 2 f1; 2; : : : ; n + 1g is xed and determine the number an (j ) of its
non-negative solutions in Zn .
448

For this we recall that (since Euler's days) such problems are dealt with
best by generating functions, namely:
E(x) = (1 + x1 + x21 + : : : )  (1 + x2 + x22 + : : : )  : : :
1 1 1 1
X
= 1 , x  1 , x2  1 , x3 : : : = p(k)xk
k=0
where p(k) denotes the number of partitions of k; that is, the number of
unordered representations of k as k = s1 + s2 +    + se with sj a positive
integer for j = 1; 2; : : : ; e, or equivalently k = 1n1 +2n2 +    + knk , where
nj  0 is the number of appearances of summand j . Therefore, all partitions
with summand j forbidden are obtained via
1
X ,1
jX 1,
X
(1,xj )E (x) = (1,xj ) p(k)xk = p(k)xk+ p(k),p(k,j )xk :
k=0 k=0 k=j
Hence the desired amount an (j ) equals:
if n + 1  j , 1

an (j ) = p(n + 1);
p(n + 1) , p(n + 1 , j ); if n + 1  j

Also solved by HEINZ-JURGEN SEIFFERT, Berlin, Germany. There was one
incorrect solution submitted.
Janous remarks how his ideas above can be extended to include the case where
the excluded summand can be a subset of the values from 1 to n + 1.

Crux Mathematicorum
Founding Editors / Redacteurs-fondateurs: Leopold Sauve & Frederick G.B. Maskell
Editors emeriti / Redacteur-emeriti: G.W. Sands, R.E. Woodrow, Bruce L.R. Shawyer
Mathematical Mayhem
Founding Editors / Redacteurs-fondateurs: Patrick Surry & Ravi Vakil
Editors emeriti / Redacteurs-emeriti: Philip Jong, Je Higham,
J.P. Grossman, Andre Chang, Naoki Sato, Cyrus Hsia

You might also like